Among all self-identified "rationalist" communities that I know of, and Less Wrong in particular, there is an obvious gender imbalance—a male/female ratio tilted strongly toward males.

Yet surely epistemic and instrumental rationality have no gender signature.  There is no such thing as masculine probability theory or feminine decision theory.

There could be some entirely innocuous explanation for this imbalance.  Perhaps, by sheer historical contingency, aspiring rationalists are recruited primarily from the atheist/libertarian/technophile cluster, which has a gender imbalance for its own reasons—having nothing to do with rationality or rationalists; and this is the entire explanation.

Uh huh.  Sure.

And then there are the less innocuous explanations—those that point an accusing finger at the rationalist community, or at womankind.

If possible, let's try not to make things worse in the course of having this discussion.  Remember that to name two parts of a community is to split that community—see the Robbers Cave experiment:  Two labels → two groups.  Let us try not to make some of our fellow rationalists feel singled-out as objects of scrutiny, here.  But in the long run especially, it is not a good thing if half the potential audience is being actively filtered out; whatever the cause, the effect is noticeable, and we can't afford to ignore the question.

These are the major possibilities that I see:

(1)  While the pure math of the right Way has no gender signatures on it, we can imagine that men and women are annoyed to different degrees by different mistakes.  Suppose that Less Wrong is too disagreeable—that relative to the ideal, just-right, perfectly-rational amount of disagreement, we have a little more disagreement than that.  You can imagine that to the men, this seems normal, forgivable, takeable in-stride—wrong, perhaps, but not really all that annoying.  And you can imagine that conversely, the female-dominated mirror-image of Less Wrong would involve too much agreement relative to the ideal—lots of comments agreeing with each other—and that while this would seem normal, forgivable, takeable-in-stride to the female majority, it would drive the men up the wall, and some of them would leave, and the rest would be gritting their teeth.  (This example plays to gender stereotypes, but that's because I'm speculating blindly; my brain only knows half the story and has to guess at the other half.  Less obvious hypotheses are also welcome.)  In a case like this, you begin by checking with trusted female rationalists to see if they think you're doing anything characteristically male, irrational, and annoying.

(2)  The above points a finger at the rationalist community, and in particular its men, as making a mistake that drives away rational women.  The complementary explanation would say:  "No, we have exactly the rational amount of argument as it stands, or even too little.  Male newcomers are fine with this, but female newcomers feel that there's too much conflict and disagreement and they leave."  The true Way has no gender signature, but you can have a mistake that is characteristic of one sex but not the other, or a mistake that has been culturally inculcated in one gender but not the other.  In this case we try to survey female newcomers to see what aspects seem like turn-offs (whether normatively rational or not), and then fix it (if not normatively rational) or try to soften the impact somehow (if normatively rational).  (Ultimately, though, rationality is tough for everyone—there are parts that are hard for anyone to swallow, and you just have to make it as easy as you can.)

(3)  It could be some indefinable difference of style—"indefinable" meaning that we can't pin it down tightly enough to duplicate—whereby male writers tend to attract male recruits and female writers attract female recruits.  On this hypothesis, male writers end up with mostly male readers for much the same reason that Japanese writers end up with mostly Japanese readers.  In this case I would suggest to potential female authors that they should write more, including new introductions and similar recruiting material.  We could try for a mix of authorial genders in the material first encountered on-site.  (By the same logic that if we wanted more Japanese rationalists we might encourage potential writers who happened to be Japanese.)

(4)  We could be looking at a direct gender difference—where I parenthetically note that (by convention in such discussions) "gender" refers to a culture's concept of what it means to be a man or woman, while "sex" refers to actual distinctions of XX versus XY chromosomes.  For example, consider this inspirational poster from a 1970s childrens' book.  "Boys are pilots... girls are stewardesses... boys are doctors... girls are nurses."  "Modern" cultures may still have a strong dose of "boys are rational, girls are un-self-controlled creatures of pure feeling who find logic and indeed all verbal argument to be vaguely unfeminine".  I suppose the main remedy would be (a) to try and correct this the same way you would correct any other sort of childhood damage to sanity and (b) present strong female rationalist role models.

(5)  The complementary hypothesis is a direct sex difference—i.e., the average female human actually is less interested in and compelled by deliberative reasoning compared to the average male human.  If you were motivated to correct the sex balance regardless, you would consider e.g. where to find a prefiltered audience of people compellable by deliberative reasoning, a group that already happened to have good gender balance, and go recruiting there.

(6)  We could be looking an indirect gender difference.  Say, boys are raised to find a concept like "tsuyoku naritai" ("I want to become stronger") appealing, while girls are told to shut up and keep their heads down.  If the masculine gender concept has a stronger endorsement of aspiring to self-improvement, it will, as a side effect, make a stronger endorsement of improving one's rationality.  Again, the solutions would be female authors to tailor introductions to feminine audiences, and strong female role models.  (If you're a woman and you're a talented writer and speaker, consider reading up on antitheism and trying to become a Fifth Horsewoman alongside Dawkins, Dennett, Harris and Hitchens...?)

(7)  We could be looking at an indirect sex difference.  The obvious evolutionary psychology hypothesis behind the imbalanced gender ratio in the iconoclastic community—the atheist/libertarian/technophile cluster—is the idea that males are inherently more attracted to gambles that seem high-risk and high-reward; they are more driven to try out strange ideas that come with big promises, because the genetic payoff for an unusually successful male has a much higher upper bound than the genetic payoff for an unusually successful female.  It seems to me that male teenagers especially have something like a higher cognitive temperature, an ability to wander into strange places both good and bad.  To some extent, this can be viewed as a problem of authorial style as well as innate dispositions—there's no law that says you have to emphasize the strangeness.  You could start right out with pictures of a happy gender-balanced rationalist unchurch somewhere, and banner the page "A Return To Sanity".  But a difference as basic as "more male teenagers have a high cognitive temperature" could prove very hard to address completely.

(8)  Then there's the hypothesis made infamous by Larry Summers:  Male variance in IQ (not the mean) is higher, so the right tail is dominated by males as you get further out.  I know that just mentioning this sort of thing can cause a webpage to burst into flames, and so I would like to once again point out that individual IQ differences, whether derived from genes or eating lead-based paint as a kid, are already as awful as it gets—nothing is made any worse by talking about groups, since groups are just made out of individuals.  The universe is already dreadful along this dimension, so we shouldn't care more whether groups are involved—though of course, thanks to our political instincts, we do care.  The remedies in this not-actually-any-more-awful case are (a) continue the quest to systematize rationality training so that it is less exclusively the preserve of high-g individuals, and (b) recruit among prefiltered audiences that have good gender balance.

(9)  Perhaps women are less underrepresented on Less Wrong than may at first appear, and men are more likely to comment for some reason.  Or perhaps women are less likely to choose visibly feminine usernames.  The gender ratio at physical meetups, while still unbalanced, seems noticeably better than the visible gender ratio among active commenters on the Internet.  Not very plausible as a complete explanation; but we should consider hypotheses that involve unbalanced participation/visibility rather than unbalanced attraction/retention.

 

Part of the sequence The Craft and the Community

Next post: "My Way"

Previous post: "Bayesians vs. Barbarians"

New Comment
360 comments, sorted by Click to highlight new comments since: Today at 6:14 AM
Some comments are truncated due to high volume. (⌘F to expand all)Change truncation settings

We should also look for specific, teachable “gateway” skills that might allow more women to participate in LW.

I remember reading some story about how women did persistently worse in a particular organic chemistry course than men did, until they added a training session explicitly teaching mental rotation (there’s a gender gap in visual/spatial abilities), after which point test scores equalized because mentally rotating the molecules was no longer a barrier, and other skills could come into play. I can’t find the webpage, though (though there’s a bit of corroboration here), so take the story with a grain of salt.

Given the comments elsewhere in the thread about gender differences in expected agreeableness, and about women being discouraged by downvotes, it sounds like one plausible barrier concerns how to have heart in the face of criticism. Maybe someone should write a post or two on process/growth vs. trait models of ability, and how to have the former. Or on how to keep in mind that people are responding to your words, not your inner soul, and that there’s some system of rules that determines their responses that you can learn to hack. Or something along these lines. There a... (read more)

Actually, I had a number of "aha" moments along these lines when I read a book called "Hardball for women". It's a book about how to explain the cultural difference of business to women - another notoriously all-male province. It really changed the way I thought about a lot of things - pointed out the alternative point-of-view etc.

There are some really great anecdotes about differences between male and female culture - which are somewhat US-centric, and very generalised, but worth thinking about.

The one I can most easily bring to mind is that in general, boys, while growing up, rough-house a lot when they play. So they learn that a bit of ribbing is just in fun... whereas a lot of girls never do - the only exposure girls have to either physical or verbal roughness is when they get told off for doing something wrong... so they learn that when it gets rough, they're in for trouble.

I recognised in myself that when my boss told me I'd done something wrong, I had a really strong negative reaction compared with most of my male colleagues. They had realised that the boss was just letting them know what not to do, so it didn't happen again. I'd automatically gone into... (read more)

[-][anonymous]11y150

Teaching thicker skin a good idea. Even a blog post on the psychology of receiving and responding to anonymous Internet criticisms and engaging in debates without taking it personally would be interesting to me.

As a woman, I suspect the people on the internet forums on which I feel most at home make an effort to be nicer to me (and other women). Whenever I comment on those forums anonymously, there are many more negative comments and they are more aggressive than any I receive when I'm not anonymous- comments both from men and women. Maybe just associating a comment with a name or a face makes people more friendly in general- I don't know.

As a person who is more motivated by criticism than praise, I tend to be careful about researching and crafting my comments to avoid unhelpful or obvious attacks, because criticisms tend to attract an inordinate amount of my attention and I'll fixate on the one criticism and forget about all the upvotes and praise. I try to keep things in perspective but it's my personality to focus more on errors.

In my experience women like to share their thoughts with everyone but can be less inclined to argue with random strangers. Depending on the topic, some of the lesswrong comment threads seem to be a forum for debate, and less of a place to share thoughts. Maybe if they were reframed as "share your take" instead of "dive into the debate," they'd have more more appeal, but I don't know if that's the goal.

7Aurini15y
Brilliant posts, Anna. Would you consider doing this?
5Robi Rahman8y
Is there somewhere I can find a comprehensive list of mental skills that men are typically worse at than women? I'm male and it just occurred to me that I probably ought to practice those.
0AspiringRationalist12y
I think teaching "gateway skills" is an excellent idea. One potential impediment to making it work here (as far as I can tell, this is still the case 3 years later) is that the Core Sequences are essentially prerequisites to understanding most of the content here. In order to successfully bring in new people with differently inclined personalities (including, but not limited to more women), posts on "gateway skills" will need to be accessible to a more general audience. This is certainly doable; it would just require a break from routine. Another potential benefit of this idea is that it may help current readers develop skills that they are less inclined to develop, and are consequently somewhat lacking in (I would consider myself part of this category).

I was talking to my brother the other day about the blinders that come from hanging out only with math/physics/compsci nerds. And he suggested that yes, it is valuable to expose oneself to many types of people, but looking for “normal people” or “non-nerds” is the wrong way to do it; normal people are boring. The thing to do is to find people who share some other kind of passionate interest -- people’s whose enthusiasm for public speaking, or windsurfing, or whatever it is has driven the creation of their own interesting, idiosyncratic culture.

As a student, I participated in a (fairly small) number of programs for women in math. The programs were all lousy. I love it when I find other women I can really talk to -- it makes me feel more at home with myself, my gender, and my ability to learn to think. But these programs weren’t like that. These programs were blah. “Adding more women” is a boring aim, like “meeting normal people” or “meeting non-nerds”. Usually it’s achieved by taking whatever it is that might make the program distinctive (e.g., math talent, or an analytical/argumentative spirit) and watering down that distinctiveness until more women are involved.

I don’t kno... (read more)

I do want to emphasize - it was in a previous version of the post, in fact, but I took it out - that I am maintaining my phrasing of my goal as create rationalists not create female rationalists. But if half of the audience is being filtered for some silly avoidable reason, then I want to fix that.

I am maintaining my phrasing of my goal as create rationalists not create female rationalists

There is a strong selfish incentive for single male rationalists to pursue this goal, though. I know I would love to have my next girlfriend be a rationalist (if only to avoid my most recent failure mode), and given the numbers, that's probably not something every male rationalist can hope for right now.

3Vladimir_Nesov15y
One point is that it's rather silly of people to filter out for silly reasons. You don't stop reading a good book because it uses a funny font. This may be made into a general warning, a failure mode to be avoided, and linked to from the introductory article. Although I understand that it's not a mode of thinking that is likely to work where the mistake surfaces.

You don't stop reading a good book because it uses a funny font.

You very well might, if you found the font so distracting that you couldn't enjoy the book. I think that you can only assert that this is a failure mode by misunderstanding who is being "silly" and who has control of avoiding the "avoidable".

[-][anonymous]15y110

You don't stop reading a good book because it uses a funny font.

Of course not! You stop reading it because there are too many plot holes, the characters irritate you or the author is just too naive for you to stomache. Meanwhile the guy who bought the other printing of the book which has a more aesthetic font keeps reading to the end and then gives it to his friends.

I had similar experiences in my first year of university (though it was Women in Science instead of Math, a slightly larger population). It was boring.

Women in Rationality screams "pointless PC navel-gazing" because of association with these experiences.

3taryneast13y
Yup, me too, but it was "Women in IT". I stopped going to that and started hanging out with the local linux group - far more interesting, despite the inevitable gender-imbalance.
6[anonymous]6y
This is what we're doing at LW Netherlands. The "partner" community we've chosen is the spirituality community, which strikes me as remarkably complementary to LW in multiple ways. We're going to weekly ecstatic dance parties, some of us are signing up for zen retreats (which is a bit more masculine), and there's the potential that some of us will try tantra at some point. And it's really gold for learning rationality, because when it comes to lines of attack on becoming smarter, spirituality couldn't be more different from, yet as potent as, our strategy. Bonus is that their gender ratio is pretty much the inverse of ours.
4A1987dM10y
(To anyone else reading this nearly five years after it was posted: one year later, Alicorn did this.)
-2[anonymous]15y
I do want to emphasize - it was in a previous version of the post, in fact, but I took it out - that I am maintaining my phrasing of my goal as create rationalists not create female rationalists. But if half of my audience is being filtered for some silly avoidable reason, then I want to fix that.
-2[anonymous]15y
I do want to emphasize - it was in a previous version of the post, in fact, but I took it out - that I am maintaining my phrasing of my goal as create rationalists not create female rationalists. But if half of my audience is being filtered for some silly avoidable reason, then I want to fix that.
-2[anonymous]15y
I do want to emphasize - it was in a previous version of the post, in fact, but I took it out - that I am maintaining my phrasing of my goal as create rationalists not create female rationalists. But if half of my audience is being filtered for some silly avoidable reason, then I want to fix that.
1[anonymous]15y
Triple post. Please fix.

I've lurked OB/LW for quite some time now (about a year) and haven't posted much for many of the same reasons as divia (intimidated by the quality, felt like I wasn't familiar enough, etc) and have tried to get a few people that are interested in this kind of thing to follow along with me to little success. This post made me wonder why people I was so sure would care about rationality didn't care to join the community here and further why I sit on the sidelines.

My first thoughts were that this group feels "cliquey". There are a lot of in-phrases and technical jargon floating around, which to an outsider can be very intimidating.

On top of that every incorrect comment is completely and utterly destroyed by multiple people. I know and you know we're dismantling ideas in an attempt to kick out biases and fallacies every time they appear, but to an outsider it looks/feels like an attack on all fronts. I think this stems from the separation of ideas from the self, which is really the first step on the road to rationality. Anyone who hasn't made that step feels like they are being personally attacked, and it isn't an easy step to make. Dislodging your ideas from your self-image ... (read more)

I think this stems from the separation of ideas from the self, which is really the first step on the road to rationality. Anyone who hasn't made that step feels like they are being personally attacked, and it isn't an easy step to make.

Even if you've made the step in general, it doesn't help when people use status-signaling language in their comments. e.g. "Have you thought of X?" is a lot better than, say, "Clearly you haven't paid any attention to X", if your goal is to actually improve discussion, rather than to get a charge from demolishing your opponent. (I suspect that the concept of a martial art of rationality doesn't help with this, from a priming perspective.)

Setting a frame of etiquette that indicates we are all here to help people become rationalists rather than to show off our own skills at rationalism might help with this.

We are engaged in a collaborative effort that produces a webpage documenting the interplay of ideas. For example

The comment does not consider X

The reply does not explain why X is important

X is important because ...

The argument for the importance of X is unconvincing because ...

The flaw in the argument is easily remedied thus ...

Addressing the commentor is a mistake. It invites the replier to read the commentors mind to the detriminate of responding to the actual words of the comment.

I'm sensitised to this from attempting to teach Go to beginners. It is Black's move that makes bad shape/is too close to thickness/small/slow. If I have to correct a mistake I don't say "your move was bad", I say "black's move is bad". Black and White are characters in a collaborative fiction and me and my pupil are having an Author to Author conversation about how to maintain the dramatic tension and not just have White beat up Black.

1dclayh15y
Off-topic, but: surely you want to teach your Go student to win, not to have a close game? As per Eliezer's favorite swordfighting quote?
2Jonii14y
In go, good move stretches as far as possible, but not further. Moving too far away from your group is just as bad as moving too close. If both players follow the flow of the game, neither can expect to crush the other, the game is symmetric. This is why you just take the board position and see how the game should flow from there. If your opponent is weak, he loses much because his moves don't accomplish enough, and maybe even actively defeat each other. You can't do anything but avoid falling prey for that same thing, making each of your move count as much as possible, being as sharp as possible. It's easiest to win against those players who have something like intention to kill. When they stretch further than they actually could, you can just lazily defend yourself. Defending is much easier than attacking in go. After a while, opponent has overstretched formations around the board, and you can start retaliating with no noticeable weaknesses, making the game totally one-sided. You just can't do better than playing the sharpest move possible. If opponent answers well, the result is even, but that's just how the game is.
-5AspiringRationalist12y
5Aurini15y
As a student, I would love to see this. As an argumentative SOB I need to consider this. As an opinionated member of LW: damnit, this is front page stuff, right here! This is bang on the money, and a hell of a lot less misogynistic than my own reactions to the post!

My first thoughts were that this group feels "cliquey". There are a lot of in-phrases and technical jargon floating around, which to an outsider can be very intimidating.

This is a feature, not a bug. If you spend a day discussing, say, Newcomb's problem, and it doesn't change the way you think and speak about similar situations in the future -- if you don't find easier, faster ways of describing the situation, which were previously unavailable to you -- then you've probably wasted a day.

The effect this has on newcomers is a bug though. Hopefully the Wiki, once it's active and fully implemented, will help to address this.

On top of that every incorrect comment is completely and utterly destroyed by multiple people.

I desperately wish that there were a way to emotionally differentiate between attacking a meme someone is carrying and attacking a person.

4MrHen15y
I don't agree with this. Maybe it is because I am new, but I spend half of my time here translating everything into a more common language. I find it easier to keep track of different arguments and reasonings once I translate it into the linguistic matrix I have been using to learn about everything else in my life. Brand new concepts need new words and terms, but Newcomb's problem isn't one of them. The term "one-box" is jargon. "Omega" is jargon. It speeds up discussion on Less Wrong, not the real world. If I translate those terms into short sentences I can begin to have the conversation with anyone and the reusable terms will bump into topics I remember from other conversations I have had with people outside of Less Wrong and I see no harm in typing four words instead of one. To beat this to death: If I always talk about Omega as "Omega," I think about it as Omega. If I think of Omega has someone who has a perfect guessing rate at what I am going to do, this reminds me of omniscience and that reminds me of how a lot of Christians view their God. Is there any relation between Newcomb's problem and God? Who knows, but it seems an interesting train of thought. If I end up talking to a Christian about Newcomb's problem and they state that Omega seems like God I have a better recourse already in place than simply saying, "No, Omega is Omega; not God." That being said, I have to register the terms "Omega" and "one-box" because I am engaging in conversations here at LW. But even if I spent all day talking about Newcomb's problem using these new terms, I do not consider the point of the conversation to have the same conversation faster or easier. Neither do I consider the point to be having similar future conversations faster or easier. Faster and easier are luxuries; they are icing on the cake. I want to learn new concepts and I consider this to be very different than learning new jargon. Backing up a little, "cliquey" holds negative connotations. In-phrases and tech
8dclayh15y
Agreed. I know that when I'm talking with philosophers I tend to use their special prepositions ("On X's view...", "Y consists in...") to sound more in-groupy and thus give extra weight to my arguments. On OB/LW this primarily takes the form (started by Eliezer, I think) of embedding a link to a previous article in every other sentence, which certainly comes off as intimidating, at least to me.

On OB/LW this primarily takes the form (started by Eliezer, I think) of embedding a link to a previous article in every other sentence, which certainly comes off as intimidating, at least to me.

That's interesting -- I quite enjoy that convention, and feel like it makes the site more penetrable to newcomers. To me, the purpose of the links seems to be "if this sentence seems to follow from the last, keep reading. If I seem to have made an unsupported leap, you may profit by following the link."

1dclayh15y
It's nice for reading, yes (although it does mean that reading one Eliezer post can quickly turn into eight tabs' worth of previous posts), but when it comes to writing a post (or even a comment), I feel like if I don't have a bunch of references I'm leaving myself open to accusations of "Oh, that point was addressed here, here, and here. Try doing some reading." Which might not be a bad thing, necessarily: it's certainly not too productive to be constantly going over the same ground as MrHen says below, but it certainly does affect what I choose to write.
3MBlume15y
I spent a lot of happy afternoons this way last year (didn't get much done on my quantum problem sets though) Ah, this I totally get. I think this might be a good function for the welcome thread -- you could just leave a comment saying "hi, I'm thinking about writing something about X -- is there anything I ought to be reading first?"
0MrHen15y
I imagine that some of this task will be handled by the wiki or the tags assigned to each post. That being said, I have little problem with someone talking about a topic that was broached seventy times previously as long as it either adds a new perspective or is a decent summary or launching point for people not there during the past discussions. That being said, having "little problem with" may mean I will not read it because I consider the topic saturated.
2[anonymous]15y
It's interesting to see how that comes across to you. When I include links one of my motivations is actually to towards less exclusiveness. Something along the lines of "I'm using this term but acknowledge that it is in group jargon. Here's the several pages of text I saved reproducing for anyone who wants it." I usually associate the in group status game with making it difficult to get information and so ensuring that you can gain status through every piece of knowledge the newcomers must aquire. Why show them where to learn stuff when you could be shooting them down every time they speak?
6pjeby15y
When Eliezer does it, I interpret it as a desire not to repeat himself. When other people do it, sometimes my first impression is that the person is implying they are better-read and more knowledgeable, i.e., that they're trying to signal superior status by implying "I have been here longer and know more," as well as implying a stronger in-group affiliation, by the amount of work they've done to dig up appropriate scriptures and link to them. The tone of the non-linked portion of the comment of course makes a big difference, of course. "Have you read XYZ? It seems to me like what you're saying contradicts point Q; how would you address that?" would be a lot different than some of the comments I've seen that look like trying to win an argument by the volume of their citations.
2MrHen15y
I think I see it as something between you two. I sometimes see it as "I agree with these articles so these articles agree with me." This probably qualifies as a weird form of appealing to authority. To make it fit better with your view, "If I put my article in a list of their articles I am like them." The charitable side of me thinks of it as tracing someone's train of thought backwards. "Oh, so that's why they were thinking about this subject."
0taryneast13y
Yup - especially when the linked-to post doesn't actually support what they were trying to say. I sometimes see it as a form of thought-stoppage.
2JackChristopher15y
I get link fatigue when read LW/OB. But I think it's unavoidable. It has to be done for at least two reasons: 1. There's a lot of conceptual "bittage". As the writer, you not only have to close the inferential gap between new concepts, but close it for every new word. That's a lot to explain (and to see, if a new reader) at once. 2. The medium of blogging wasn't designed to visualize information of this depth. And that means heavy link back.
0MrHen15y
Yeah, my philosophy classes had a lot of people who would skip over discussions by using a well-known name. This is similar to what Andrew's You don't need Kant post was talking about. That being said, the other extreme is not terribly useful, either. I have trouble remembering philosopher's names because the arguments and logic are more interesting and I never bothered associating it with the person who was speaking. As it turns out, I spend a lot of time going over ground that has already been covered because I did not learn the shortcut term. This could be seen as a counter-point to my comment above.
2MBlume15y
I didn't actually mean that it makes it easier to talk about Newcomb's problem, more that if, say, we're talking about the Israeli government dealing with a hostage situation, and someone says the Israelis should "one-box," they mean to communicate that "not only the effects on the current situation, but the impact their decision-making process will have on others trying to predict their actions, should be salient to their decision"
3MrHen15y
Funny, I would not have associated one-boxing to mean what you described. I assumed that it only really matters when dealing with a perfect predictor. Apparently I missed some form of "action implies predicability" side of the discussion? In any case, looks like I get to go do some research/thinking. Thanks. Ah, okay. Thanks for the clarification. I think my points more or less stand as is but could probably have been less targeted at the Newcomb's problem example. (Topic branch) Something I personally do dodge issues in terms is to rotate synonyms throughout a discussion to troll for bad assumptions in terms. If anyone gets a "Wait, what?" look on their face it means we may not be on the same page.
1MBlume15y
That is an excellent idea.
3Paul Crowley15y
The wiki is entirely ready to go; all it needs is more contributions.
2matt15y
We have a styled and integrated wiki under development, but it's on the same platform as the current wiki - we'll pull in all content from the current wiki when we finish. Full support for ciphergoth's sentiments from the devs - go forth and enwiki the good stuff.
0Paul Crowley15y
Will it cause you any trouble that not all users have the same username on the wiki as they do here?
0matt15y
We may look into it later, but we're not currently looking to merge user accounts. LW is Python, MediaWiki is PHP; the database formats are different; we don't generally love working in PHP; etc. Both will continue to be open source projects (of course), so contributions will be welcome :).
0Paul Crowley15y
I think MediaWiki has some provision for supporting SSO, but yeah, I don't fancy hacking on it myself and I imagine you have better things to do too!
0Vladimir_Nesov15y
It also needs a long-term hosting, so that you can safely link to it, and not worry that the target URL will go away or get abandoned.
0Paul Crowley15y
Another solution would be to support wikilinking in the markup language; that way if the wiki moves the links can move with it.
0Vladimir_Nesov15y
This is too hard to enforce outside the wiki, where anyone can fix failures to conform.
0Paul Crowley15y
The greater convenience of the wiki markup might be enough, though?
0gwern15y
I think ameliorating that issue is one of the main reasons for the Less Wrong wiki. Is it helpful in even its current state?
2Paul Crowley15y
We could add a "Jargon request" category, so if you want to know what a term means and Google can't tell you, create a page on the wiki and just put {{pagewanted}} in there, and then I or someone else might notice and fix it. EDIT: I've done this.
-2SforSingularity14y
These apply to both genders...

There are a number of average sex differences in personality traits that would contribute to more males identifying as "rationalists" than females.

Here are the sex differences found in the Big Five personality inventory, from a cross-cultural survey by Costa et al.:

  • Women score higher on Agreeableness

  • Men score higher on the Assertiveness facet of Extraversion

  • Men score higher on Openness to Ideas, especially in the US. Women score higher on Openness to Feelings and Openness to Aesthetics. In the US, men also score higher on Openness to Fantasy.

  • Some particular items, such as identification with the word "logic," were skewed strongly towards males

An interest in rationality may depend on Openness to Ideas. Otherwise, someone just isn't going to care about the kind of things we talk about here.

Furthermore, the identification of males, but not females, with words like "logic" suggests that perhaps part of the gender gap of interest in rationality is about words like "logic," and "rationality." Women are often labeled as "irrational" or "illogical" when they are perceived as overemotional, and this labeling m... (read more)

Do women, on average, have more connected social lives than men do? It's very easy for a few people with no life to effectively dominate a community like this simply by spending more time than any "normal" person would want to. If women are more likely to have "a life" and less likely to become fixated on a specific hobby, that could explain why we see fewer women commenters. (One reason I'm here is that I have very few people in Real Life that I talk to regularly.)

A possibly relevant data point is that males are roughly four times more likely to have autism or Asperger's syndrome than females.

6Alicorn15y
May or may not be connected, but I do have Asperger's.
1taryneast13y
My dad has Asperger's. I have some of his traits... serious introvert (need a lot of time alone), can't deal with too much stimuli (light, music, social situations) ...but I'm actually pretty good at the "recognising emotions from faces" tests, so I tend to test negative.
3Vladimir_Nesov15y
I don't believe that any significant portion of this community has these conditions, so it's not a relevant data point.
9MBlume15y
This is, to me, a non-obvious claim. (For example...) That depends on whether you consider autism or Asperger's to be discrete states, or to be extremes of traits which may be found to a lesser extent in individuals labeled neurotypical. If the latter, then gender distribution of autism/Asperger's could be relevant to discussion of the milder versions of those traits
1byrnema15y
I was considering the consequences to the gender ratio if it is true that LW draws from people who are nerdy and social. It seems that "nerdy" qualities tend to be associated with men (perhaps due to correlation with autism traits), and social skills tend to be associated with women. While plenty of men have great social skills, even nerdy men, what fraction of nerdy women have good social skills? From my experience, women in math and science have a good chance of not feeling socially comfortable. While men have a higher chance of autism traits, I wonder if within the sub-population of math and science, women have a higher incidence.
0Vladimir_Nesov15y
However you interpret my message, these factors can't significantly account for male/female participation ratio, as I'm pretty sure they don't concern at all, in any form at least 70% of the community.

I think MBlume's point was that there is a fairly mainstream theory of autism spectrum disorders (which includes Asperger's) that claims they can be explained as extreme cases of the 'male brain'. If there is a correlation between the male brain traits that in extreme form are diagnosed as autism/Asperger's and the patterns of thinking that would lead to an interest in this community and if it is true that autism/Asperger's fall on a continuum rather than being discretely identifiable conditions then the gender bias observed here could be explained by the same factors that explain the gender bias in these conditions.

The implicit hypothesis here is that the average community member on this site would score higher on tests designed to diagnose autism spectrum disorders than the general population, without necessarily scoring high enough to be diagnosed with the condition. That seems at least plausible to me.

2Aurini15y
This suggestion accounts for women being underrepresented, but not for their distinct absence (unless if several popular posters are, in fact, female).
2taryneast13y
Um, Anna Salamon... Alicorn... I don't count as "popular" but those two are top 10 karma-wise, (Alicorn is third). I'm sure there are others. There is a severe under-representation, but we're not entirely absent :)
8Aurini13y
2 year old comment. They may not have been as prominent back then (in fact, I think that was the post that made gender an issue here). :) I'm trying to get Girlfriend into this site, because that will make her sexier.
0taryneast9y
They were prominent then too (I was there then too) :)
0A1987dM10y
BTW, 8.4% of respondents to the 2011 survey were female, compared to 3% in 2009.

A few thoughts:

(1) I agree with Nanani, and think it would be awful to actively try to "recruit" females, or even really do anything to entice them to come/stay. Though I appreciate the spirit of the post nonetheless because I think it's a very interesting and important issue, and I think it's okay to acknowledge it and question it. If anything, efforts to even out the male/female imbalance would have to be made on a much greater scale to start to see change.

(2) Do people really think that it's an issue of females frequenting Less Wrong and then LEAVING? I doubt it. I suspect that a much lower proportion of females even happen upon the blog in the first place. This would eliminate a number of the explanations.

(3) This is an issue that deeply intrigues me. I have some fairly simple theories. Unfortunately, I am not well-versed enough in evol. psych., gender studies, history, sociology, etc. to feel like I have enough background to really get at the heart of the matter. So most of my ideas are purely anecdotal.

I believe that females on a whole are less interested in intellectual pursuits. Particularly intellectual pursuits that are HARD and take a higher amount of mental ho... (read more)

7MBlume15y
I distinctly remember my first meeting with one of my female friends, she was staring at a poster on the wall which explained why e^(i pi)=-1, copying down each step, and clearly trying to understand it. This was not in connection to any class, she was just interested. And I remember being immediately, strongly attracted to her simply for that reason, because of that demonstrated, genuine curiosity. Which indicates that on some level, I perceived that trait as being remarkable, though I'm not sure that that's specifically because she was a girl. (For those looking for the end of the story, my best friend was already actively pursuing her (which is why we were being introduced), and I chose to respect the friendship.)
2[anonymous]15y
Fascinating. I can somewhat emphasise with that having as I do the exact reverse experience. I haven't managed to create a habit of telling myself to think about trivial things while spacing out but I have done so with regards to (at least an acceptable fraction of) social encounters. It seems that thinking up trivial things is a distinctly non-trivial task! How ever do you manage it? This site is full of cute fluffy puppies. I agree and obvserve that projecting the apearance of thin skin is a somewhat more effective gambit for females than males.
0Alicorn15y
I'm the most intellectually curious person I know (in non-Less Wrong circles, anyway), but of course I could be an exception.
2[anonymous]15y
And that serves to remind me that my default image of posters is skewed slightly too far in the male direction. It takes a kick in the face by an unambiguous implication in the post body to make me even consider that the author isn't a relatively young male with a science, engineering or programming background. I had finished Alicorn's post and got up to (2) in Michelle's before I was prompted to check the author's name for gender association.
2Alicorn15y
Considering that an alicorn is a unicorn's horn, I think mine is a fairly girly username. Unless there is a unicorn-loving male element I should be aware of.
4BethMo13y
Interesting... all the places I've seen the word, it meant a winged unicorn*. But reading this post drove me to look it up, and I did find both definitions. Less Wrong: raising new interest in definitions of mythological creature parts! :) *Speaking of mythological definitions, I learned somewhere to distinguish between an alicorn, which has the goat-like body, lion's tail, beard, etc. of a unicorn, vs a horned pegasus, which has horse-like features. Not sure where that came from, but it's firmly implanted in my stores of useless knowledge.
0DanielLC12y
So, unicorn pegasus actually is a meaning of alicorn? I always thought that was limited to the My Little Pony community.
0Alicorn13y
Fantasy authors are not as a general rule inclined to adhere so rigidly to your taxonomy ;)
3mattnewport15y
Apparently sufficiently girly that I didn't even know that's what it was...
7mattnewport15y
The downvote suggests I need to elaborate. Alicorn thought she was fairly clearly signaling her gender by using a feminine username. I had seen her username in previous comments and did not know the word so it did not signal her gender effectively to me. Perhaps my vocabulary is just inadequate but if I'm at all representative then I think the misunderstanding is worth noting as one small way in which male and female posters may fail to communicate due to hidden assumptions.
2Pavitra10y
Wacky theory: it sounds masculine because it ends in a consonant.
0taryneast13y
Sorry alicorn... I thought you were a guy too The nick isn't girly enough for this girl to pick up on. May be a cultural reference that's not common enough?
1SforSingularity14y
If high intellectual curiosity is a rare trait in males and a very rare one in females, then given that you are here this doesn't surprise me. You are more intellectually curious than most of the men I have met, which is itself a high intellectual curiosity sample.
0taryneast13y
I don't think you're the only one. I'm one of the most curious people I know (and I'm a girl)...
0rabidchicken13y
I would not describe myself as curious, but my brain automatically creates a few absurd theories per day, and I go nuts if I don't test them all. (which generally means I spend half an hour on wikipedia, and then repeat the process tomorrow with the new data)

Far from a complete explanation, but it often is hard to simultaneously view oneself as female and as intellectually able, even given evidence of intellectual ability. Role models can help, but artificially manufacturing role models (e.g., by preferentially making women’s writing visible) has its own costs. Others’ remarked surprise at how one is at once female and intellectual/rational/etc. can make this harder.

One relevant subskill here is... I don’t know how to say it. Something like “the ability to keep in mind the whole complex layout of the evidence, without letting your anticipations get overwhelmed by the nearest cliche”. So that even though gender is terribly salient (more salient than, say, GRE scores), gender doesn’t affect one’s views of one’s abilities to a greater extent than do similarly informative non-gender data points.

A second relevant subskill is the ability to put in a full effort even in the presence of threatening stereotypes and probable failure. Eliezer has written about many aspects of this one, but not the “in the presence of threatening stereotypes” part.

If anyone feels up to writing a tutorial on one of these skills, I'd like to read it. And it might be useful to both members of underrepresented groups and everyone else.

Remember that to name two parts of a community is to split that community

Gender's far from the only division here, I would say. There's also a difference in approach to rationalism, that may also have some overlap with gender differences.

I personally consider myself interested in rationalism for the practical benefits: models that are useful, for real-life definitions of useful... not useful for "Knowing The Absolute Truth And Being Right". However, this doesn't appear to be a common attitude on LW.

In the computing field, there's a stereotype that says the difference between men and women is that men care about computing for its own sake, whereas women care about doing other things with computers, how computers can be used to interact with people, and so on. In other words, that women have a more instrumental view of computers than men.

Of course, some men take this to mean that women are therefore not as skilled as men with computers, but I have not found this to be true. The women I've known in computing were happy to develop as much skill as was required by their instrumental aims -- quite often more skill than the men I knew! They just didn't make a religion o... (read more)

Strongly seconded. To sum up the most important points:

  • Instrumental rationality has its own skillset, related to but far from identical to the current OB/LW corpus. It's a skillset we need if we want to deal well with the practical world.

  • Right now, folks with skill at instrumental rationality who come upon LW are likely to leave again. We aren't set up to give them what they're looking for, or to avoid misinterpreting them, or to ask for what they can teach us.

  • Adding a partial focus on practical, visible applications (i.e., including instrumental rationality in LW) might well improve the gender balance.

0NancyLebovitz13y
I'm pleased to note that there's rather more instrumental rationality at LW (though perhaps not so much recently), with AnnaSalamon having contributed a good bit of it.

If you really truly want to improve the gender balance on LessWrong.com, you will delete this post from the sequence, and never bring it up again. I know it's well-meaning, but as a woman it just makes me feel weird and singled out. I am convinced from long, sad experience that as long as the conversation circles around gender, it'll do more harm than good; I find the research on stereotype threat to be powerfully convincing and explanatory.

In reading through the comments (I didn't get to all 240 of them, I'll admit), I found it striking that constructive suggestions occurred when someone reframed the question from "How can we make women feel more welcome?" to "How can we make newcomers feel more welcome?"

And to everyone who was so ready to come up with biological/evolutionary theories to explain this possible gender imbalance, I have a stunning, heretical statement to make:

Men's and women's brains are not significantly different. Observed differences between genders in thought patterns and behavior are cultural and can change.

I humbly suggest that if you disagree, then read the actual studies that make claims to the contrary. Ask yourself, "How emotionally... (read more)

I have some conjectures.

1) People tend to hold beliefs for social reasons. For example, belief in theism allows membership of the theist community, the actual existence of a deity is largely irrelevant.

2) For most people, in order to maintain close social relationships it is necessary to maintain harmonious beliefs with nearby members of your social network. Changing your beliefs may harm your social ties.

3) The larger your social network, the more you have to lose by changing your beliefs.

4) Less Wrong encourages questioning and changing of beliefs.

5) On average, women have larger social networks than men.

6) Less Wrong encourages the adoption of strange and boring beliefs, largely based in maths and science.

7) Advocating strange and boring beliefs does not signal high status, rather it signals a misunderstanding of widely accepted social norms, and therefore poor social skills.

8) Much of a woman's percieved value as a human being is tied to her ability to navigate the social world, men may be forgiven for making the occasional faux pas, women are not. Women are therefore strongly averse to signalling poor social skills.

Some predictions:

1) Willingness to join Less Wrong is inverse... (read more)

9MBlume15y
Rationalists should win, and human beings need social networks for emotional well-being. Is it possible to * hold true beliefs * be honest about those beliefs * make friends and keep them? In my experience, my atheism, for example, has not been a huge handicap (with one glaring exception), but it's certainly hurt me from time to time. People feel that if nothing else, their beliefs deserve "respect," and I have learned no graceful way of indicating that I have given long consideration to the matter, and give their beliefs no greater probability than I do Santa Claus or the Harry Potter novels, without giving insult. This would, I think, be an art worth learning. (The closest, I think, I've ever come, was by saying that "where I come from," the way you give respect to a belief is by actively working to see if it's true or not -- which often simply means attacking it; that "in my culture," an attack on a belief is a sign of deep respect.)

I find myself prefacing a lot of statements with "Where I come from" or "On this side of the water" when I'm talking to a religious person whose friendship I desire to keep e.g. my parents. This lets you provide exactly the same argument, which probably ends up being processed in exactly the same way, while letting the other person know that you don't expect them to assent immediately.

2Tom_Talbot15y
I think the answer to your question may be no. I've thought on my original post some more and realised that I made a mistake in number (8), one cannot signal poor social skills since signalling is a social skill (it serves no other purpose), a person who cannot signal optimally is a person with poor social skills. So if a tendency towards telling the truth disrupts a person's ability to signal optimally, then rationality and popularity must forever remain opposed, since in order to be rational you must give up your ability to signal popular, false beliefs. Even if we say that "rationalism" is only believing the truth - you can lie if you want to - your ability to signal is still disrupted, since the most effective way to signal is to sincerely believe what you're saying
8taryneast13y
I'd re-address point 8 by saying that a women is rewarded socially for subverting her own beliefs at the expense of those of her social group. She is actively punished if she steps away form group-norms (eg by pointing out errors in the groupthink or common misconceptions) - starting by getting a "boy you're weird" look, glances amongst the others to indicate that "they all think you're weird" and other social pressures. If you persist, this can go to the "polite pulling aside" - where usually one of the women will explain to you that you are being disruptive (usually by couching it in "we're really concerned for you" language)... and then on to hostility, usually involving a heavy dose of "you're not respecting our opinions"... finally to shunning/ostracism from the group. Women learn pretty quick that you either put up or shut up... and that if you "don't have anything nice to say, then don't say it at all". The exceptions I've found are mainly amongst girl-geeks, SF-fandom and the other usual haunts no doubt familiar to all here... which also are nearly always predominantly male.
1CronoDAS15y
I suggested something similar above; participating in an online discussion forum, such as this one, is a time sink that competes with maintaining an offline social network. If is true, then there will be more male commenters, because women have better things to do than waste time commenting here.

I am reminded of Paul Graham's explanation for the low number of female startup partners from Ideas for Startups:

I didn't realize it till I was writing this, but that may help explain why there are so few female startup founders. I read on the Internet (so it must be true) that only 1.7% of VC-backed startups are founded by women. The percentage of female hackers is small, but not that small. So why the discrepancy?

When you realize that successful startups tend to have multiple founders who were already friends, a possible explanation emerges. People's best friends are likely to be of the same sex, and if one group is a minority in some population, pairs of them will be a minority squared. [1]

I would suspect that all the more fundamental reasons (2, 4, 5, 6, 7, and 8) are factors, but that they are then magnified by 1 and 3. As far as 9 is concerned, I am female myself and have never commented on Less Wrong before, to provide a single, anecdotal data point.

8AnnaSalamon15y
Any idea why you haven't?

Not entirely sure, though I believe I did post a couple of comments to Overcoming Bias a while back. I used to comment on reddit and comment semi-regularly on Hacker News, which refutes the first explanation that I thought of, that it was a matter of my time, since clearly I do sometimes take time to comment on the internet.

The comments here are high quality, which is somewhat intimidating, and also makes things take longer, since I want to think more carefully about what I say, but that would probably apply to Hacker News as well.

A possible explanation consistent with the quotation I mentioned is that even though I read all the posts here and on Overcoming Bias, I don't think I've thought about the issues deeply enough to have much original to contribute. And that may have something to do with the fact that most of my friends aren't all that interested in the topics. I imagine if I were talking about the posts more often in real life I would feel like I had more to contribute.

I'm in a similar situation - I comment (sometimes) on reddit and HNews, and have occasionally posted a few sentences to OB, but I am much less likely to comment here. The high quality of the posts and comments leads me to agonize a bit overmuch about every part of a comment, and sometimes I will write, edit, and rewrite a comment before deciding to just not comment at all. I, too, often feel I would not be contributing anything original.

(I should also note in this comment that I am male.)

6taryneast13y
Well, I'm female and I agree with what you say. I often get the feeling that I'm barely well-read enough to follow a conversation here, and the comments I make are only on side-issues, or ones that I have experience of from "the outside" (eg IT or on being female). I've made a few witty quips and minor points elsewhere... but they really aren't part of a full discussion. I get the feeling that I am a complete and total novice (not a problem), and that I need to have at least read all the way through all the sequences (million words or thereabouts wasn't it?) before I can even get around a lot of the nuances brought up by the other commenters... and if I try posting before then, I'll get it wrong, get some rather swift kicks in the premises (which are a downer even if well-intended) and feel less likely to stick my neck out the next time... there's an awfully steep learning curve here, and it feels very hard to break in unless you're still suffering from serious overconfidence bias ;)
3rabidchicken13y
I lurked on lesswrong for about a year, because I used to be worried about losing karma and looking like an idiot. I guess I got used to it after enough terrific failures. If you want to appear consistently intelligent, this is a very hard site to do it on (even after you do the research)
1Tripitaka13y
I feel absolutely the same way but contribute it partly to having english as only second language. Although I am told to have a better grasp of it than most of my english-speaking peers, the high-level-concepts and language need a lot more time, and formulating original, coherent answers is even harder. I am male.
5Psy-Kosh15y
Huh, that minority-squared effect is interesting, but I'm not sure it need apply here. It'd be individuals coming here, right? It doesn't take a group to, well, come to LW. Or am I misunderstanding your point in some way?

Network-effect makes a big difference too. After all - you have to arrive here some way - usually by being told about it by friends. Sure, some people arrive by accident - just happen to be browsing through HP fanfic or something... but a lot will arrive through their friends... and a lot will stay because they find friends.

...which leads us back to people's friends tending to be same-sex. If there are few people of your own sex in the group then it's got less... er... ambient friend-potential...

you have to work harder to be with a bunch of people that have a different culture than yourself. Genders have different cultures, so add that on top of the new culture of LW itself and unless you're a particularly socially-capable person (and LWs tend not to be), then it's less likely that you'll find friends.

Obviously this is a generalisation and likely only a very small part of the pressures involved in a very complicated process... but it's there.

4divia15y
While it's ultimately true that individuals come to LW, not groups, I'm far more likely to follow and especially to comment on blogs that my friends also read. For me, one primary way I get really interested in subjects and motivated to understand them well is by talking about them to my friends in real life. And most of my friends are girls.
8Psy-Kosh15y
hrm... actually, I'm reminded of something. Several years back, someone designed these simulations that basically ran an algorithm like "assume people don't mind being around people that are different, so long as at least some small fraction of their nearest neighbors are also like themselves.", and basically simulated people moving around to fulfil those criteria. The simulation would consistently produce highly segregated results. Aha! here's a site with applets that run such simulations: http://www.dartmouth.edu/~segregation/segregation-simulator.html
3badger15y
Just an short addendum: Thomas Schelling is the one who originally thought up this model.
3gwern15y
I think the point is that there are multiple factors which all reduce the chances. In startup founding, you need multiple similar people; in LW browsing, you need multiple personal characteristics. Maybe 90% of women can handle the disagreeableness; maybe an independent 90% can handle the male-style-writing; maybe another 90% is unswayed by cultural gender differences; maybe another 90% are unaffected by a female genetic predisposition against reasoning (I'm just running down Eliezer's list), and so on. A LW commentor who is female would be in the subset of women who is in all these groups. (Just with these few factors, we're down to something like only 60% of women are 'eligible' for LW membership to begin with!)
2Psy-Kosh15y
Sure, that all makes sense, but an LW commentor who is male would also have to fall into multiple subsets. The question isn't "why are so few members of the total human population on LW?" but "what's with the different proportion of males and females?"
6gwern15y
No, remember that our various sets are already biased towards males (obviously males don't mind 'male-style-writing'). The point of my comment is that a few small biases can quickly multiply up. If on all these factors, the males are at 95% where the females are at 90%, then we only need like 10 factors before we would expect twice as many males than females based just on those factors alone and ignoring any feedback or network effects. Why we mostly have male-style-writing, or why there might be a female genetic predisposition against reasoning, are all different issues one would expect different answers to. (That there are such gender differences isn't too terribly surprising to me, personally - finding that males and females are exactly the same on all these factors would be like finding that all of humanity is 100% genetically homogeneous, and that there's no truth to, say, the Japanese having a low tolerance for alcohol or some groups being lactose-intolerant or Africans being disposed to sickle-cell anemia.)

I'm a female-type person. I can't speak to anyone else, but I did make a post a while ago, and it was met largely with indifference and I wound up taking a (small) karma hit. This did a variety of things, some useful and some not, but one thing it hasn't done is encourage me to take the time to write another top-level post.

If I'm wandering around a large in-person gathering and I drift over to an interesting conversation and say something and get shot down - even if it's because I said something stupid - I'm more likely to drift away or at least shut up rather than continue to hang out with and seek approval from Those People Who Were Mean To Me™. "Drifting away" is much easier on the Internet, and if more women are giving up after making one or two poorly-received comments, that could easily explain the gender bias.

Possible solutions if I have the right idea (no idea how palatable they are to others):

1) Be more parsimonious with downvotes and generous with upvotes in general.

2) Attempt to draw out individual women Less Wrong ers on particular topics (solicited input puts one out on less of a social limb).

3) Identify who makes each vote on a comment or post, so people can identify Those People Who Were Mean To Me™ and not have to consider the entire Less Wrong community as a whole to be united against them.

I wonder if there is a gender difference in tone of the way people introduce themselves to a group. Per my experience, the girl way seems to be personal sharing (signal: "I'm approachable"), the guy way seems to be chiming in on topic (signal: "I'm capable"). Since your article was weighted more to personal sharing than to providing something topically useful, I think you might have gotten a confused reaction from the regulars ("how is this supposed to help me be a rationalist?").

I wonder if allowing explicitly flagged "hello / about me posts" would help? Normal contextual politeness would kick in and the response to such a post would be much less aggressive.

1Paul Crowley15y
Regular open threads for introductory posts?
4MBlume15y
how's this?
3byrnema15y
I really like the newcomer welcome page. I also really like what ciphergoth and others are doing with their self-introductions; explaining when and how they came to LW, and a little about their perspective and what their goals are in LW . While this is a big step in the right direction, I think it could go further to be a lot better. When a newcomer comes to LW, a warm welcome consists of two parts. First, they introduce themselves to you; this is the welcome page. Second, an introduction from you should be available to them, at the click of a button. When you first arrive at LW, it feels like a huge dark opera hall of masked voices. It would be great if whenever you read an interesting comment, you can click on that person's name and read their self-introduction. The problem with the welcome page as it is currently built is that it would difficult for a newcomer to retrieve introductions over the weeks or months that they are getting to know us. I don't know how Reddit works, but it just occurred to me that one simple solution would be to make comments written on the welcome page special so that they're always listed first in the list of comments, regardless of when it was written.
2AnnaSalamon15y
I like it :). Two small suggestions: 1. You might consider changing the phrase "please feel free to leave a comment" -- it might be more welcoming to just ask people to "please leave a comment", giving the impression that we want to hear from them. (Though I'm not confident this would be better.) 2. It would be nice to invite questions, not only on LW jargon (which you do), but on the etiquette of posting and voting, why those mean people may have downvoted one of your comments (and why you shouldn't take that to mean we won't appreciate you), etc. I'm not sure how to gracefully incorporate this into your text. Eliezer should add a link from LW's "About" page. (Except, the link should move somehow with the month's welcome thread, if we have new ones every month. What's your plan here?)
4MBlume15y
hmm, here's a spiel on voting and karma, I'm a little worried that it sounds too forbidding -- what do you think?
2AnnaSalamon15y
My non-confident impression is that it's good. One slightly friendlier suggestion would be to replace "Don't be discouraged by this." with "Don't be discouraged by this; it happened to many of us." Thanks for building us the welcome thread.
0MBlume15y
I like that No problem =)
0MBlume15y
1 Done =) Is "We'd love to know who you are, what you're doing, and how you found us." too strong do you think? 2 Good idea -- let's see what I can come up with.... Yeah, I'm not sure how well this works with the one-a-month structure. Ideally I'd like this thread to be stickied to the front page, but I know that requires some admin-help.
0[anonymous]15y
hmm, here's a spiel on voting and karma, I'm a little worried that it sounds too forbidding -- what do you think?
1AnnaSalamon15y
Good idea. With the threads for introductory posts linked to from the (to be built) welcome page, and with newcomers encouraged to introduce themselves and ask questions.
1MBlume15y
I just had a go at an introductory/welcome page. Any suggestions? welcome
0Vladimir_Nesov15y
The monthly Open Thread may be repurposed to also act as a more fleshed-out introductory & welcome thread.

I'm not sure I like your solutions but I think your sort of experience might not be atypical. My female friends and family have often reacted to criticism of their ideas with what I (a man) found to be an overly defensive posture. My reply was always to tell them not to take things so personally. My guess is that boys are tend to receive more encouragement and confidences boosting from parents and teachers and so are more confident putting their ideas out there and don't take poor reception as hard- but I don't really know.

I've definitely made comments (here and elsewhere) that were taken poorly and lead me to back off commenting for a while. I know where your coming from but I think identifying votes can easily lead retaliatory voting which is all kinds of irrational and is a disincentive for honest voting. I'd also be wary of devaluing karma by being more generous with it.

I'm curious what you have in mind for (2). I guess if topics were specifically about gender-related biases there would be room for it. I think some of few women here might be annoyed by this.

My suggestions are two fold.

  1. It would be nice if there was some information on individual comments regarding either the

... (read more)
6MBlume15y
While acknowledging that we're talking about a small sample size here, this matches my experiences -- especially in the area of religion. Agreed. This can be time-consuming -- it's a good ideal, but we should not have a norm of down-votes requiring an explanation. Cannot agree enough.
0MrHen15y
I wonder if you are subconsciously more aggressive in the area of religion. Another explanation would be that religious women are inherently more defensive.
5Eliezer Yudkowsky15y
Explaining downvotes for newcomers (as shown by join date) would economize on effort where the marginal payoff is high.
1MBlume15y
I'm trying to think of a simple icon which could appear by user-name in comments to indicate either "I have been an active member for <X weeks" or "I have posted <X comments". My first thought was a cartoon of a newborn, but that seems a bit patronizing. ETA: Ideally the icon would be the same height as the username itself, which doesn't give us many pixels to play with.

Why not just when you click Vote Down, if they're considered new, a little message appears that says " is new to the site. Could you gently explain why you are disrecommending their comment to others?"

3MBlume15y
I like this
3taryneast13y
Lots of sites have this kind of thing - and the commonest implementation I see is "five whatevers" (eg five stars or five coffee beans or in our case five paperclips?) where they start out grey and progressively get coloured-in to indicate... not time-since joined but a combination of that and of active participation in the community (usually numbers of posts and replies). We could easily compare time-joined to karma points. EY et al would get five paperclips, a newbie with no karma would start with none. The paperclips could work on a logarithmic scale.
8TheOtherDave13y
At which point the natural desire to earn status within communities would drive many of us to maximize paperclips. Which would be funny.
3billswift15y
Why not use what we've already got and use their karma score? Maybe show it when you mouse over the name or something?
0Alicorn15y
Maybe some average karma-per-comment/post number, rather than an absolute karma number, would skew slightly less in favor of people who have high karma scores half for sheer volume?
6MBlume15y
Well, we're trying to signal whether you should treat a particular commenter gently. If a particular commenter has posted 1000 comments, and none have been voted up, there's no need for kid gloves.
0mattnewport15y
I've seen a few forums where a user's name is accompanied by a 'rank', often humorous, indicating standing in the community. I'm not sure whether this is generally based on number of posts or length of membership or some combination of the two but it might be apt here. I'm sure someone else can do a much better job of coming up with ranks than me but something along the lines of: neophyte, aspiring rationalist, follower of the way, master rationalist, etc.
4Alicorn15y
Or in keeping with the martial arts theme, a series of belt colors? I know this varies from art to art and dojo to dojo, though.
0Dreaded_Anomaly13y
It's a while afterward (and it does not seem that this idea caught on), but I think the obvious choice would be to use the EM spectrum. Describing Eliezer as a "gamma ray rationalist" seems quite fitting to me.
-1Vladimir_Nesov15y
I think our tribe is small enough, and blatant mistakes made by commenters are rare enough, for senior members to be able to recognize the new members simply by memory, checking the commenting history on the user pages when in doubt.
2William15y
But if the tribe expands?
0wedrifid12y
We worry about any problems that brings when they happen. (Premature optimization is usually a bad idea.)
0wedrifid12y
By 'rare enough' do you mean "only about 1 in 3 comments" or is my standard of "blatant mistake" stricter than yours? (I was under the impression that you were actually more fussy than I since you mentioned being wary of hitting your downvote cap despite being in the same karma ballpark as I.)
4Vladimir_Nesov12y
The problem with Nesov_2009 is that I'm prohibited from downvoting him by the site rules.
-2wedrifid12y
Hey! How did I end up here? Must have been a bump somewhere in the recent comments.
0taryneast13y
It's not the senior members that I'd be worried about... but, say, myself. I have already made one mis-application so far - where I thought somebody was making a rookie mistake, but they actually had been around for a while and he was very upset at my correction. I think we don't have too much problem at either end of the scale, this sort of solution would help the mob in the middle.
1AspiringRationalist12y
Getting upset at being corrected sounds like a rookie mistake to me.
2PhilGoetz15y
If you click on "Preferences" under your name in the upper-right corner, you can check the box "Make my votes public".
0Alicorn15y
Where can we see the votes of people who have ticked this box?
0PhilGoetz15y
After experimenting with it, I don't think you can. Looks like it's unimplemented.
0MrHen15y
Does that preference affect which posts you have up/down voted?
0billswift15y
Thanks, I didn't realize about preferences, you solved several problems for me.
2Nanani15y
Please don't do 2 and 3.
2Alicorn15y
Why in particular don't you like those ideas?
8Nanani15y
For 2) It reeks too much of the navel-gazing "women in X" boredom occuring in education that AnnaSalamon pointed out in her comment. I certainly don't want my ideas and imput valued because of my chromosomes; I want them to be valued if and when they have merit. For 3), anyone who thinks the entire community is against them based on one negative reply has insufficiently thick skin to deal with the internet in general. The burden of effort not to think this way is on you, not on the community. If it helps, assume the mean person was just that, a Mean Person. Also, be Awesome so that anyone who is Mean to you will look stupid in comparison. Overall, I just think that encouraging niceness is just going to be more trouble than its worth, and a turnoff to participating in the community for the already-interested nerdy set that doesn't much care for such things.
5Alicorn15y
There's no reason male Less Wrong ers couldn't be drawn out individually in the same way; I only phrased it that way to keep it germane to the topic. If we had individual profiles on which we could sum up our relevant interests/activities, for instance, I could put in a little non-intrusive box "I am writing a paper on why the Reflection Principle is stupid for school" and somebody interested in the Reflection Principle could say "hey, Alicorn, do you feel like crossposting the précis of your paper here on Less Wrong? I'd like to read it." I'd be more comfortable sharing something like that at someone's request than I would just posting it on my own initiative, but there would be nothing stopping someone else of any gender from being solicited to make another post on another subject.
1MBlume15y
strongly agree with 2, only partly agree with 3. There is such a thing as efficient niceness. This isn't kindergarden, and you don't get a big gold star for 'participating'. Still, it shouldn't be a crime to post a few words acknowledging a good point, encouraging someone, or wishing someone well. Even among us guys, who are conditioned to pretend we don't need them, such practices can help keep people motivated, and keep people coming back. tl;dr: rationality/honesty should not be compromised for niceness' sake. Niceness is still possible, and indeed desirable within these constraints.
1Jack15y
I overreacted a bit. Sorry. (Edited)
0[anonymous]15y
That sounded somewhat harsh.
0[anonymous]15y
Yeah, my page hadn't loaded the rest of the thread. I didn't see someone ask why and answer. Had I I would have held off. Still.
1pjeby15y
Strongly seconded. This would be particularly helpful in discounting systematic downvoters.
6billswift15y
Much more than finding out who voted what way, I'd like to see the total upvotes and downvotes on a comment. It would be very useful to know if I got 5 upvotes and 5 downvotes or if the comment just sat there getting nothing. I'd much rather know how many people found it interesting or useful than who didn't like it. The original comment also wasn't thought through - if the "community as a whole to be united against them" occurred they'd get trashed, not a few down votes. Off-thread: I recently up voted a comment with -7 votes, because I thought it was worth reading even though probably wrong.
1pjeby15y
Oh, I'd love that too, I just want to know who the person is who logs on once or twice a week and systematically downvotes everything I posted since the last time they were on. Something like a "5 points (10+/5-)" display, linked to a page that displayed the votes would be nice. I'd contribute it if I could afford the time to really dive into the codebase and learn how it works.
0Simulacra15y
Something like reddit commentroversy would be nice as a feature of the site. Sadly it doesn't work on LW, maybe I'll try to look at it and figure out if there is a possible hack to getting working until (if) the feature is implemented here. A random comment I selected to show what it looks like [Username Changed]: username 70 points(+184/-116) 7 hours ago[-] If anyone uses reddit and doesn't have this get the greasemonkey add-on then go back to the commentroversy and click install.
0pjeby15y
A very quick bit of troubleshooting shows that the json load doesn't appear to be occurring, i.e. the $.getJSON apparently doesn't work. Edit: to be precise, the script bombs out when trying to do anything with '$.getJSON', which perhaps is not available in LW's version of jQuery? Edit 2: Ah, LW doesn't use jQuery. It uses Prototype. The script would have to be converted. But it does indeed appear that LW exports the needed data in JSON form.
0MrHen15y
I think another advantage a +x/-y display would be that sympathy votes or outcome skewing would be harder. If I see a post that is rated -7 and disagree with its status, should I vote the comment up? What if the post was -1? Would that change my vote? I think +5/-12 is harder to sympathize with than -7. I have a strong opinion that votes should be independent of each other.

Eliezer,

You once responded to someone's comment by writing:

"It would seem we don't appreciate your genius. Perhaps you should complain about this some more."

I'm a professor at a women's college and when I read this comment I thought to myself that a significant percentage of women who read this would not want to participate in this site.

http://lesswrong.com/lw/7i/rationality_is_systematized_winning/4zp

I am male with Agreeableness probably at least as high as the average female, and that comment annoyed me also. I wouldn't say that such dismissive sarcasm is never deserved, but I don't see how that post came anywhere near deserving it. Eliezer seems to have a short fuse with some individuals, but without knowing the history between them or being interested in digging it up, such comments seem mean-spirited. They may also look like an evasion.

2[anonymous]15y
I've noticed that 'Agreeableness' seems to be closely related to tollerance for 'tit for tat' responses. Ironically this means I often find the most 'Agreeable' people altogether disagreeable rather frequently.
2astray15y
It is an answer short on patience, but it was a comment short on insight. In response to a post relayed in short as: 'The common definition of rationality is stupid. Here is a new proposal that is a basic tenet of most of my writing. (Implicitly, keep this in mind when you see me talk about rationality.)', the poster simply added 'Well, I think the original definition of rationality is right, and I've said this before.' The inciting comment seems just like the responses (on Fark, HNews, etc.) to Pullum's article about Strunk & White- people who like what they learned flatly deny any counterargument.
-4Aurini15y
Have you read the original post? Yudkowski had his reasons for being hostile. Whether or not they're valid is debatable; but since that's not what you're debating I'm calling you out. Accusing somebody of 'seeming' mean spirited is BS. Either he was mean spirited, or he wasn't. Any level of 'agreeable quotient' is inadequate reason to make ad hominem attacks, while simultaneously refusing to provide evidence. If the 'seemingness' of his hostility was sufficient reason to scare female-types away - then point that out! Say: "I did the home work, and his hostility was/wasn't justified for reasons A,B,C,D, but regardless, it intimidates female-types, and the solution is X." The half-assed, poorly thought out, emotional post you just made undermines the entire purpose of this site, and is degrading to women; it implies that the curvier members of our species are incapable of admitting failure, and must be cottled and comforted any time they make a whoopsie. Grow a pair. [meta]Well, that was a delicious bit of irony.[/meta]
3HughRistik15y
Yes. Here is what Timtyler said: The thread he was referring to is here. I agree with his characterization that nobody had conclusively refuted the Wikipedia notion of rationality; Eliezer hadn't even responded to it in that thread. Tim was raising a valid criticism, and then added another post saying that he had pointed it out before and that it was unaddressed. Eliezer responded substantively to the first post, and gave the response under discussion to the second post. I agree that Eliezer had reasons to be hostile, yet I do deny that they justified such a level of hostility, which is why I hypothesized that his response was due to a short fuse with the particular individual since I know that there is a weird dynamic between them. It would have been enough to say something like "I saw your post, so you don't need to keep bringing up the Wikipedia definition of rationality. See my response above." As the person advancing a definition of rationality, I think that the onus is on Eliezer to defend it from all comers. And as the owner of the website which is trying to build a community, it may be advantageous to reign in hostility even when justified, and try to always take the high road if only for reasons of impression management. Or, in my subjective opinion, his comment was mean-spirited, while granting that others, such as you, might disagree and think that the comment was deserved. My reference to my level of Agreeableness is so that others can have context to interpret my impression. (e.g. "Well, Hugh is just bothered by Eliezer's comment because he is high in Agreeabless") This is an incorrect usage of ad hominem, since Eliezer was not making a factual claim, and my impression of his comment as mean-spirited is not implying that he is wrong. I think you are engaging in the common confusion of ad hominem and personal attack. It is neither an ad hominem nor a personal attack to characterize someone's comment as mean-spirited; note also that I was calling the
4steven046115y
From where I'm sitting this whole conversation is surreal. Yes, Eliezer said something uncivil once and was downvoted only to -4, but on the whole the tone here has been less hostile than most of the internet by any reasonable metric.
2mattnewport15y
While you make valid points, I can't help feeling that some people are missing the fact that Eliezer's comment was funny. Maybe not everyone was amused but I think it would be a shame if the occasional flippant/comical remark was not welcome here. I couldn't help but think of this which just makes me chuckle.
0[anonymous]15y
Exactly!

Agreed. This reminds me of an anecdote a high school teacher once shared with me about when he switched from coaching the boys' track team to the girls' track team. He didn't adjust his coaching strategy at all and in short order had a fair number of crying high school girls on his hands.

3AlanCrowe15y
I've found this comment too elliptical to be helpful. I've left to guess why "a significant percentage of women who read this would not want to participate in this site." Here is my guess. A comment whines of unappreciated genius, the reply is a sarcastic put down. That is not just a person-male interaction. That is a male-male interaction. I expect stereotypical female readers to tune it out as boys-will-be-boys bullshit. It is noise, so it makes the signal to noise ratio worse, but it is also tuned out pretty automatically, so it is not a large enough deterioration to drive any-one away. So my guess doesn't work. I fail. Shrug.
3Eliezer Yudkowsky15y
(And the said comment was voted down to -4, the threshold at which comments (by default) become invisible.)
1Zvi15y
That comment was so far outside what I expect to see on this site in general or from Eliezer in particular that I didn't initially realize that he was being hostile. On any comment board I've seen more than a glimpse of outside OB/LW I doubt I make that mistake. There's almost always room for improvement in this area but this is one place where I think we do an admirable job. If this were a primary cause, I would expect most forum-based sites to have the same problem, usually far worse than we do. Is there a general gender imbalance in most forums?
0Paul Crowley15y
I'm bothered that this is voted up so high. Perhaps EY's words were intemperate, but this feels more like taking the opportunity to have a jab at him than sitting down to address the broader problem he raises.
5SoullessAutomaton15y
The point is valid that, probably for reasons of social training, more women than men are likely to be turned away by such snide criticism. However, it's not clear that that sort of tone is even remotely common in comments here, and in fact the negative score Eliezer's comment received is strongly indicative that the community as a whole disapproved.
2dclayh15y
I hardly think -4 (now -3, apparently) is indicative of wholesale disapproval. Of course, if most people keep the default viewing threshold and don't click through often then it' would be impossible to tell. But since this very thread has a comment at -8, and I've seen several below -10, I don't think that's the case.

Personally I'm really annoyed by all the complaints about gender imbalance in so many smarter-than-average communities. There is high male to female ratio on almost every possible extreme of the society, both "good" extremes and "bad" extremes. This is natural. Until rationality hits the mainstream, it will stay this way. If it hits the mainstream, it will automatically balance itself. That's all.

Regardless of whether the current gender imbalance is natural, some aspects of rationalist community and of rationalist activism might work better if we could get a more even gender-balance, all else equal.

0billswift15y
Off-thread; but I hate the phrase "all else equal" in the real world all else is never equal. I think we need to try to decide what people are trying to say with the phrase and come up with a clearer way of saying it.
3Alicorn15y
"In a hypothetical situation with no confounding factors"?
7spriteless15y
Does this mean when one says 'we need more females' they mean 'we need to be more mainstream?'
3AspiringRationalist12y
My intuition says that improving the gender balance would help us become more mainstream; more diverse groups look less exclusive/threatening, so people feel more inclined to join them. Does anyone know of relevant research that would support/refute this hypothesis?
5Jack15y
This is really intriguing. Do you think this is the case because of greater IQ variance in men or is there something else?
6taw15y
There is greater everything variance in men, not just IQ. To say it crudely women stayed with the tribe, played it safe, and reproduced this way - median success was close to mean; while men took part in one big tournament, where the winners had much higher reproduction rates than losers - and median success was much lower than mean, playing it safe was like half losing.
6taryneast13y
Taw - the women also got half of their genes from the men who won those games. How does this affect the way that men behave, but not women? I am aware of research (eg the visual wall stuff) that male babies are more likely to take risks than female babies... but I'm not sure that your example gives the whole picture. Can you expand on it a bit?
8taw13y
Genes can easily act differently based on gender, and do it all the time, there's nothing remotely surprisingly about it.
1Tripitaka13y
I, too, find myself sceptical about a lot of the claims about fundamental brain-ware-differences between men/women that are often made here.I rarely see sources & credible studies linked. May I ask for reading material?
1taryneast13y
Did a quick google for it and can't dig up anything easy to reach. I'm remembering stuff from first-year psych that was basically a decade ago now. The only keywords I recall are that it was a "visual wall experiment" that was done with crawling infants (ie mainly pre-language). I'll describe the experiment in case anybody else recognises it and can point us at better references. I can remember watching the video, where infants were placed on a glass tabletop - underneath which was an obvious drop off, visible through the glass. ie the infants weren't in actual danger of falling - but it looked (to them) as though they might. The drop-off went down about a metre and was painted with a grid-pattern so the infant had clear visual clues of what it was. A reward (toy? food? can't remember) was placed at the other end of the table, and the infant could go get it by crawling across the glass, over the visual drop-off. I do not recall how many infants were in the study - but it produced a clearly distinct average gender-difference in the likelihood of whether the infant would brave the scary-looking crawl to go get it. The take-home conclusion was that males were more likely to risk more for the reward, whereas females were less likely to do so.
4jimrandomh13y
IIRC, the keyword for that experiment is "visual cliff", not "visual wall".
0taryneast13y
Aha! thank you :)
0taryneast13y
Ok, so now that we've got the correct key-phase ("visual cliff") I see that there's heaps of research using this apparatus, and most of the studies are on development of depth-perception, or infant reactions to maternal prompting etc... Can't seem to find anything on the study that I remember. Sorry.
2Jack15y
So down voting me for asking a question is a little weird.
1MBlume15y
Yeah, I thought the same thing.
1SforSingularity14y
Upvoted. I came to exactly the same conclusion. Men are extremophiles, and in (7), Eliezer explained why. As to Anna's point below, we should ask how much good can be expected to accumulate from trying to go against nature here, versus how difficult it will be. I.e. spending effort X on attracting more women to LW must be balanced against spending that same effort on something else.

Unrelated to gender, but related to inclusion: should we make LW, or some portion of LW, more accessible to teenagers somehow? It's been argued that we'll the best rationalists will be people who learn it young; but to judge by introductions in the new welcome thread, and by responses to the current survey, we seem to have few to no teenagers.

...

This is a very old thread, but I would still like to comment to make the point that I had assumed for a couple years (seriously, years) that, like so many other places on the internet, "open to anyone" actually meant "open to anyone over eighteen". And then I had assumed that I would make an embarassment of myself here, like I did some years ago on the good old sl4 wiki.

Seriously, you want us to come along with our /argumenta ex silentium/ and all? ...if this is really the community sentiment I have to wonder why the "popular Harry Potter fanfiction" angle isn't being milked more for its recruiting potential. I suppose that's what dignity is.

5AnnaSalamon13y
Are you under 18?

Fifteen right now, a sophmore at a magnet high school. Quite shallow (for instance, my biggest concern right now is my upcoming Haruhi Suzumiya cosplay). Too emotional (my AP Computer Science teacher makes me cry twice a week). Pitiably immodest (see aforementioned gratuitous reference to AP Computer Science and AP BC Calculus). I fooled around on the sl4 wiki when I was about twelve. Some people might still remember that.

Thanks for letting me know. If you want any help charting a good education, especially a good rationality education, I'd love to talk to you (I just sent you a PM also to that effect, which you can see by clicking on the red mailbox icon next to your name).

Are there any other teenagers on LW who care to reveal themselves?

More than a month too late, but I'm fifteen, and also a girl. Got here from Harry Potter and the Methods of Rationality, which I found out about from TV Tropes. You really should milk that, you know. :)

8Perplexed13y
Any suggestions how? A question that might trigger some ideas: When you first started looking around this site, what did you see that you found appealing, and what did you see that made you not want to stick around? Oh, and btw, Welcome to LessWrong.
9Perplexed13y
OnTheOtherHandle replied to this (below). Unfortunately, she is still not sure how the site's various buttons work, so it got sent to me as a PM. I'm pretty sure she wanted it shared, so I am doing so. If you want to upvote her ideas, I suppose you will have to upvote the grandparent. Here is what she wrote: Hi, um, I got your message from the envelope-shaped button on the sidebar. I don't even know if it was private or on the comments, but I can't seem to access that page to write a reply there, so I'll just send you this. Sorry if this isn't the right way to do things, it takes me a while to navigate a new site. As for ideas, well, those are difficult, so I'll start with what attracted me to LW. As I said, it was Methods of Rationality that brought me here, and what I liked about that was it was very scientific and logical without being The Spock - without shunning emotions as wrong or illogical, something I never really got. It made me laugh out loud many times and even cry once or twice. Because of this, it worked as a story first and foremost. If a piece of fiction is overtly trying to promote a philosophy, then it earns HUGE bonus points for actually being a good story in its own right. I'd say it served as a nice, fun way to "ease into" the rationalist community. Plus, it really made me feel for the transhumanist cause, and made me think hard about the idea that death was inevitable or acceptable, even though I can't say with certainty yet that I'm a transhumanist. As for what made me stay, well, this is fascinating stuff. I love science and psychology. I was already an atheist and a huge nerd and had already read some Dawkins and Feynman before seeing this site, so I guess that helped me to not become so overwhelmed by the sheer amount of material here. I think a little background is important, because even though LW is pretty accessable, I can't say it's for beginners. (But of course, since I've been randomly article-hopping, it's likely that I missed t
6shokwave13y
This is a reply the comment that got accidentally sent to Perplexed. I, too, got that feeling when I first browsed here. My solution to it was to look at the karma of comments: if the edit: comment got up above 5 karma, it probably is a reason why the post is wrong. If the edit: comment was at neutral or negative karma, it probably isn't a reason why the post is wrong. I don't have much data on how effective this method is, but it seemed to work for me. That's not a general solution, though. The general cure to looking immature might be to further promote a community norm of resolving disagreements. I see many arguments ending with one person admitting/realising they were mistaken (this is something that I had never seen anywhere else on the internet), but I see more disagreements left hanging. I think if we saw lots of disagreements with the posts, and most of them were resolved by reading through the comment tree, new people would see disagreements being resolved on the internet and be very impressed!
1wedrifid13y
Are those comment/post substitutions what you intended to say? I was initially confused but that correction made sense of it. That policy seems to be a reasonable one. I use approximately the same interpretation except on topics that get political. "Correctness" becomes much less correlated to karma in such cases. Resolving disagreements on the internet is impressive, isn't it? People just stopped being wrong on the internet! WTF? At the same time there is a place for leaving things hanging. Sometimes leaving aside disagreements without making a fuss or engaging in status battles can be good enough. Particularly in those (frequent) cases where the issue isn't cut and dry. When there is merit in both points but this can't be simply acknowledged without reconstructing and translating from not-quite-compatible models of reality. I bother to mention this because I've noticed that sometimes trying to resolve differences can at times do more harm than good, despite the best of intentions. Some people actually get offended if you try to be conciliatory, bizarre as it may seem.
0shokwave13y
Yeah, thanks for correcting. And I definitely agree, disagreements are not universally solved by one party admitting fault - but I do feel that there are a large number of cases on LessWrong where I don't know if one side made a mistake, or if the case doesn't have a right answer, or whatnot. And I would like to see more of those cases, the ones floating in between, to be resolved in either one direction or the other.
0anon89513y
Part of the problem is that any attempt at direct enforcement or pressure could deter people from commenting in the first place, knowing that if they did they'd be expected to see any disagreements through to the end. (That's been mentioned in previous threads, I think.) Random thought: Would individuals trying to shift the norm by setting an example work any better? Like, one person going through their comment history (possibly using the link here), and making a list in their profile page of unresolved disagreements and their current status (possibly including otherwise unvoiced ones), plus a list of resolved disagreements and how they were resolved, or a list of posts and comments that led them to shift their beliefs (incrementally or otherwise) on something? Not volunteering either way, though. In the past I've occasionally killed time reading my old posts on forums, and on reading regrettable things I've tried to fix them by amending them in replies or putting notes about them my profile, but that doesn't seem like the same thing. Basically, it seems you(general) would need to make a deliberate effort to continue discussions even after it becomes pure work, because you value having a site where disagreements are resolved more than you value anything else you might be doing with that time. Edit: I idealistically hope that when agreement is impractical, people who try long enough can still reach a better level of understanding than the standard "agreement to disagree" cliché.
0wedrifid13y
I know what you mean. There isn't always time to go and do the research oneself on each topic so as to judge between the positions.
0rabidchicken13y
(responding to perplexed's copy) When I came here from MOR, I just started by reading the sequences one by one, and then started going through new posts whenever they came up, and it seemed to work fairly well. There have been a couple of posts on the site which I did not completely agree with, but I still learned something from all of them, and it helped me get into the habit of updating my beliefs more easily. Also, I am glad that there are other people my age out there who are interested by sites like this :p
0[anonymous]13y
Welcome aboard!

I am 18 years old, somewhat new to LW and not as congruent/rational as desired. If your offer stands for others who are not under 18, I'd love to hear about that. I could use some help.

I do not think there is a need make LW more accessible to teenagers. I am not even sure I know what that means. Are we, those younger, that alien? (rethorical question) "Teenage" is a joke. Not as funny as religion, though. (A 12 years old is sitting at a bar with coworkers after a long day of work, 150 years ago: normal) As far as a I am concerned the differences between "teenagers" and adults are from age segregation. Differences in style, not principles.

That said, I am new. To the Welcome page!

8rabidchicken13y
Upvoted. I am always surprised how widespread discrimination against teenagers is, considering that everyone has to be one at some point. Every difference between people below the age of majority, and people above which an adult has been able to point out to me when I have discussed this seems to be a product of the culture we are raised in, not an inherent quality of humans within an arbitrary age range.

You might be interested in reading The Case Against Adolescence by Dr. Robert Epstein. (I believe the last edition is called Teen 2.0)

It is eye-opening on many aspects. There is a story in it that struck me. A twelve year old had an affair with his married teacher (who had two kids on her own). She went to prison for two-three years. Once out of the slammer, she had sex again with her ex-pupil. This time around 7-8 years of prison. While in prison she gave birth to a child. The child was raised by the father (the student) and that father's mother. Reporters asked the young man if the imprisoned woman had abused of him; he answered negatively, that love united them. Once the ex-teacher got out of prison for the second time she married the then adult lover, went into their car with their kid and rolled into the sunset.

How immature. Both of them. Love at teenage? Meh. Love is only for old people like Romeo (~16) and Juliet. (~14) Those crazy homo sapiens.

4rabidchicken13y
12 seems too young to me for a sexual relationship, but that may be due to social conditioning. On average, males become capable of reproduction when they reach 12-13. (which probably is also when they become capable of enjoying it) If he was willingly involved there should not be an issue, but the general assumption seems to be that if someone in early puberty already is having sex, (particularly with an adult) they are being threatened or coerced into it. To make matters more complicated, I have heard of cases where children willingly were in a relationship like what you mentioned, and then condemned the adult under parental pressure. the flip side of the argument for protecting children is that since older teens and adults can also be forced to have sex against their will, how can we remove restrictions at any age? having stronger muscles and a bit more experience is of limited use if someone threatens you with a gun. I don't know if I can actually come to an opinion on whether we should have an age of consent to shelter children, (even the ones who may want to have sex) or assume that anyone who has reached puberty is mature enough to have a relationship, and tell someone if they are being abused. I probably have not collected enough information at this point :p
3IanV13y
There are driver's licenses how about sex' licenses? (minus the minimal age requirement) You have to show your ability to have sane sex through a written test and a practical test. Or something.
6Alicorn13y
Ummmmm.
2shokwave13y
Much like the job of ruler or king, anyone who wants the job is disqualified.
0Vaniver13y
My hope is that means "correct use of contraceptives." My problem with a 'qualification' to go through a rite of passage is that the rite of passage is what turns you from unqualified to qualified. When can we trust someone to make decisions about sex? When they're not a virgin. And so we either need hard control (no sex until your wedding night with the socially approved partner) or little-no control combined with acceptance of mistakes (regretting your choice for your first time is commonplace). The idea that a licensing agency could differentiate between people that will make good and bad choices about sex is as ridiculous as the idea that a licensing agency could differentiate between people that will make good and bad choices about driving. The future is too uncertain to have good predictive ability.
1rabidchicken13y
Errr.... I'm an not sure how exactly that would work. People already are taught how to use contraception from an early age, (maybe not young enough though), and the rest of what you need to know appears to be instinctive. My question was how could you have a country with no age limit for consent which does not become a refuge for child predators. If a child wants to have sex with someone older, it should not be an issue, and if the child is forced into it, then it is rape whether there is an age of consent or not. So as long as children are encouraged to speak out if they are being forced into sex, having an age limit may be unnecessary. Nobody here has actually argued in favour of an age of consent yet, which I find interesting. Does it have any benefits compared to only punishing adults who have been proven to act without a child's consent?
0wedrifid13y
One option would be to allow it but require that the younger individual specify the desire publicly in a suitable manner.
1wedrifid13y
Well, technically... ;)

I am a 16 year old. To be honest, most teens wouldn't handle the site. The requirement for an understanding of mathematics, logic, and science are beyond the reach of most, and the desire of most of the rest. That said, I have introduced two friends of mine to HPMOR and they have taken to it, and I am leading them towards Less Wrong. On the other hand? I don't know how many adults would handle less wrong either. If you want my advice on how to be more appealing to teenagers, it is relatively simple.

Link everything, so that someone who doesn't understand can follow your links and find out. Useful more for teens than for adults, it is still good practice. Few intelligent teens will tolerate a teens area for long.

0Viliam_Bur13y
The same could be said about most adults.
9hamnox13y
I don't feel quite comfortable admitting that I am only 18 over the internet. (But I'll do it anyways, obviously.) Irrational fear of internet predators is irrational >.> It's hard not to feel a little intimidated by the sheer sanity of what's written here. For a long time I felt like I was obligated to at least get my GenEd done before I could sign up to comment. If I haven't managed to pass society's standard of intelligence yet, how can I expect respect and understanding here, where the standards are so much higher? There's probably quite a few teen lurkers out there, waiting hopefully for some small sign to inform them when they are high enough on the sanity-waterline to converse with gods. Edit: Oh yes! And I'm a female. Slightly relevant to the original posting :)
5Randaly13y
I'm 16.
4Solvent13y
(insert standard creepy late post disclaimer...) I'm 17, but have been reading LW for more than a year (and telling all my friends to do the same, of course.) I think that at least for the smart, nerdy, sciency type teenagers I hang out with, LW isn't too scary to get into. I could certainly manage it. It was a bit hard to get into, though. If I didn't love Three Worlds Collide and philosophy so much, I probably wouldn't have bothered. All the "initial reading" that LW provided at the time felt to me like the worst of Eliezer's output: the Simple Truth, and so on. The first truly awesome post I saw on here was the one which introduced "Shut Up and Multiply" to my vocabulary. And I think I might PM you about that good education, hoping you extend that offer more generally.
4benelliott13y
I'm 17, if anyone's still looking.
3NaomiLong13y
This is a little less than a year late, but oh well. I'm an almost-18 year old female who found LessWrong through HPMoR, which a friend of mine recommended to me (he is also interested in LessWrong and regularly reads the site). If you see this, I would love any advice you have to offer about "charting a good education, especially a good rationality education."
0Michelle_Z13y
Same! A good rationality education is extremely interesting to me. I would love to hear more information about that. I am also an 18 year old female who found Less Wrong through HPMoR, here, Naomi.
2Larks13y
I'm still a teenager! I think I've already mentioned it, but maybe not.
2rabidchicken13y
I am male and seventeen, started reading LW when I was sixteen after being directed to H:MOR from TV tropes. I recommended MOR to a few friends and they enjoy it, but don't seem interested in rationality as much as I am. I generally find the site accessible and to have material which is easy to understand, but still teaches me new things regularly. I am working through the sequence about quantum physics right now, since the major sequences listed as 3.1-3.4 + 3.6-3.8 did not take long to read through. also, seeing "disagreements being resolved on the internet" is honestly one of the most inspiring parts of this site to me :p
2peuddO13y
One of the reasons why I took the step from lurker to user - a month or so ago - was that I thought I should reply to this comment. I subsequently forgot where to find it, and stumbled upon it again just now. I'm 18. Whether or not that makes me qualified for whatever help you had in mind I do not know, but I'm certainly interested.
0EStokes13y
Hi, I was wondering if you had any advice on education for LW'ers outside the US? edit: That is, general education advice that applies to people even outside the US?
-3RHollerith13y
I'd take her up on that, kid :)
2Eliezer Yudkowsky13y
Don't call her "kid", grup.
2RHollerith13y
Definition from Urban Dictionary: "A grown-up in the minority, finding themselves among younger people. Coined by Adrian Spies, writer of the Star Trek episode featuring a post-apocalyptic world where only teenagers survived." Which fits because I am on the far upper end of the age distribution here (which Eliezer knows because I attend meetups in the Bay Area).

I found LW as a teenager and it seems extremely accessible, at least to me, but nobody in my age group who I've shown it to has agreed.

3Michelle_Z13y
Very old, but I wanted to post that I've found the exact same issue. I find it accessible, especially when I do some research, but so far no one I know has shown even the slightest interest. I've mentioned it to two friends, both of whom I consider very intelligent, but neither took the bait. My female friend loved HP MOR, though.
0KPier13y
I'm another data point (16 when I first started reading) who found it very accessible.I've tried a couple different ways of getting my friends interested, with varying degrees of success. I think the problem with getting most people (regardless of age) interested in a site like this is that you have to expend a lot of time even to understand what everyone's talking about, and it doesn't seem worth it "just for a blog".
0MBlume15y
If this can be done without significant compromise, it definitely should be.

This is an old post and I have little to add, but I notice that I'm very surprised and a bit put off by it. I'm surprised and put off by many similar things.

I speak as a female with an intuitive grasp of logic, anal-retentiveness and detail-orientedness. I also have primarily made friends with neurodiverse people, with a disproportionately large percentage being on the autism spectrum. I almost became a standard computers-and-stuff-that-xkcd-talks-about geek, but ended up becoming a video-games-and-anime-and-fanfiction geek instead. In an alternate universe, I might have eventually ended up a technical writer or computer programmer.

And as such, it always feels strange and off-putting to me when people talk about how there aren't women on Less Wrong (/playing video games/whatever the "masculine" pursuit of the week is) and speculate about how these mysterious, socially-oriented creatures are put off by the most attractive qualities (e.g., people not being agreement-bots). I'm probably exceptionally prone to thinking that I should go away because nobody wants me here (or at whatever other place it is), but things like this make me feel unwelcome. But so does everything else... (read more)

I've only just come into contact with this place, and normally I avoid commenting the day I start somewhere, but this post was compelling considering how I found LW.

A very good friend of ours sent a link to LW to my husband, but not to me. Usually he will send links to both of us he believes we'll both be interested in, and links only to me that he feels I'll be interested in but not my husband, and vice versa.

So clearly he felt I wouldn't be interested in this place, despite knowing that I am fond of rational discourse. Fortunately, my husband knew I would, and so I am here. I just found it an interesting data point in the context of this particular conversation.

Edit: Though this makes me wonder, why didn't I come across LW myself? Why didn't I bother searching for such things?

3PhilGoetz15y
Interesting point: How does anybody find LW? Suppose you're out in cyberspace, wanting to discuss rationality. What search term could you enter to find this place? Googling "rationality" doesn't turn up LW. Should I put a link to Less Wrong in the Wikipedia page on rationality? Is there a better keyword than 'rationality' for LW?
6taryneast13y
My sister sent me a link to HP:MOR I read it, got to the end, and decided I wanted more MOR... So I dug into it. EY's username is "lesswrong" which I googled and found this site. As a side-point. My sister doesn't seem to have been interested in coming to lesswrong. I also posted the link on facebook... so all my friends (male and female) would have seen it. I've also blogged about it on my tech-blog (I have 175 regular subscribers - mostly male but some female through girlgeek blogs). No idea if anybody followed me through the looking glass.
4Nanani15y
How about linking to it on TV Tropes? Clearly many participants of LW are Tropers already. Tropers are young, nerdy, and numerous, after all. Also less likely to be put off by dense linkage throughout articles. Such as This one for some Weird But True topics, or This One for the Weirdest topic of all. After all There Is No Such Thing As Notability
4gwern15y
Please don't. As a long-time Wikipedian, I can tell you any sane editor will nuke that addition on sight becuase it looks like (and IMO, is) self-promotion. More logical places to add it, where it might make sense, would be the pages on Eliezer Yudkowsky and SIAI. (I'd add Robin Hanson, but I get the impression he's chosen to remain only associated with OB. What's up with that anyway? Sometimes it feels like LW/OB is a schism - the EY-ites have migrated to LW, and the Hansonites squat on the remnants of OB.)
4PhilGoetz15y
I've migrated to LW because threaded discussions are SO VERY MUCH BETTER! Locating old posts is easier; commenting is quicker; karma is fun (and a rationality test - not the getting it, but the not caring too much about it). EDIT: The ability to edit my comments is also a huge win. I always write something wrong the first time, even when taking this rule into account. Also, if I catch up on my reading and make 3 comments, on OB I have to wait an hour before I can make a 4th comment.
0SoullessAutomaton15y
The latter is of debatable relevance; and it's already linked from the former (and has been for weeks, it seems).
1swestrup15y
Well, as an additional data point on how folks find less wrong, I found it through Overcoming Bias. I found that site via a link from some extropian or transhumanist blog, although I'm not sure which. And I found the current set of my extropian and/or transhumanist blogs by actively looking for articles on cutting-edge science, which turn out to often be referenced by transhumanist blogs.
1CronoDAS15y
I, too, found Less Wrong from Overcoming Bias; I'm pretty sure I found Overcoming Bias from some comment on author David Brin's blog, but I don't remember when.

(This looks pretty old, but I decided it couldn't hurt to be the Female with an Anecdote)

I'd found Less Wrong when I was already looking for a better understanding of rationality than could be found browsing through random atheist blogs, so I pounced on the sequences like a rabid kitten. When I went looking for how to actually apply the general principles of rationality, my mind naturally gravitated towards, well... Its own functioning. And the ways I wound up applying what I learned were substantially less about the 'calibration' and 'winning' that had first caught my eye.

I came for the dissent, like a good Intellectual Hipster, but I think I stayed for Luminosity.

It's not true to say that I just don't have a great personal interest in abstract epistemics, or winning, or making sure that my beliefs are correct, because I do. I really, really do. But as soon as I calmed down from Man-With-A-Hammer-Syndrome, I found that I don't like straight-up arguing nearly as much as I thought I did, though I absolutely stand by the necessity of sharpening our minds against each other. I enjoy pieces on how fully rational people might interact with others more than I like the more abstract musi... (read more)

As a rationalist who happens to be female, here is my take on this:

1) On an ideal amount of agreement vs disagreement : while it may be true that female dominated segments of the internet have much more agreement in their comments than male dominated ones, these same segments are significantly less rational, on average, and to a degree so are the topics they revolve around.

Rationalists tend not to bother with stating the obvious, and there isn't much "nice post" type commentary around here, so even if the amount of agreeing were higher on this community, it would not be obvious. This "invisible agreement" issue has been discussed before isn't really all that tied to gender as far as I can tell.

2) Can't comment on this because obviously, LW and OB do not contain significant turn offs for me.

3) If a recruit is attracted because the poster shares their sex organs, they aren't a very promising recruit.

How about an experiment where a male writer posts under a more feminine name?

As for recruiting Japanese rationalists, good luck doing that in English. Maybe some of your key posts ought to be translated instead. Hire a professional.

4) Agreed.

5) Sad, but probably ... (read more)

9MichaelVassar15y
I have my own issues with armchair evolutionary psychology, and to a much lesser degree, with Eliezer's armchair evolutionary psychology, but he said nothing very rationally questionable here IMHO, and certainly nothing that "The 10,000 Year Explosion" (well written but not very persuasive on most of the claims that I didn't already agree with and occasionally flat-out poorly reasoned) would call into question.
2taryneast13y
Yup. In fact I recall, in the early days of the internet, pretending to be male most of the time. I found it annoying to be continually hassled by a mob of lecherous boys when I just wanted to kick back and blow of steam bogging about on the local MUD or with a few rounds of networked doom. These days, of course, it's different. For one thing - even here there is a considerably higher proportion of women that are actually likely to be women IRL. :)

Perhaps, by sheer historical contingency, aspiring rationalists are recruited primarily from the atheist/libertarian/technophile cluster, which has a gender imbalance for its own reasons—having nothing to do with rationality or rationalists; and this is the entire explanation.

This seems immensely more likely than anything on that list. Libertarian ideology is tremendously dominated by white males - coincidentally, I bet the rationality community matches that demographic - both primarily male, and primarily caucasian - am I wrong? I'm not big into the rationalist community, so this is a theoretical prediction right here. Meanwhile, which of the listed justifications is equally likely to apply to both white females and non-white males?

Now, that's not to say the list of reasons has no impact. Just that the reason you dismissed, offhand, almost certainly dominates the spread, and the other reasons are comparatively trivial in terms of impact. If you want to solve the problem you'll need to accurately describe the problem.

-2[anonymous]11y
Conversely, if some here see it as a feature and not a problem, it would be in their interest to deflect attention away from this cynical and oh-so-unfair explanation. Really, isn't it impolite to suggest that some people might dread acommodating different backgrounds and perspectives that threaten the neatness and comfort of an exclusive community? Any drive to become less exclusionary is basically a moral signaling spiral, a kind of creeping social justice activism! It corrupts and dumbs down everything it touches, it has no use for our brilliance and fearless dissent!

Whatever it is that makes women less likely to participate in sites such as Less Wrong, I am completely oblivious to it. For whatever reason, a high percentage of boys would find Less Wrong boring, and so would an even higher percentage of girls. It is true that my everyday-life interests are more "feminine" than seems to be the LW average (writing fiction, composing music, singing in a choir, as opposed to hard-sciences math and physics, which I chose not to study in university partly for the reason that my teachers wanted me to because I'm a gi... (read more)

1). There is a lot of, for want of a better term, "mental masturbation" around here: arguing for the sake of arguing, debating insignificant points, flashy but ultimately useless displays of intellect etc. Men tend to enjoy this sort of thing much more than women. Perhaps the female equivalent would be "social masturbation" -- endless gossiping about other people's trivia.

2). There's a major bias toward discussing math and science topics on here, and objective rather than subjective experience. Rationality, as a meta-construct, arguably... (read more)

Conspiracy theories tend to be male dominated, much more so than LW is. Yet the anti-vax conspiracy seems to be female-dominated and cater heavily to females.

One explanation is that the issue has to do with children, meaning that it appeals to parents and general and mothers in particular (for, so sue me, evo-psych reasons).

Nonetheless, it would be interesting to study that conspiracy theory and see what sort of other effects occurred indirectly from any original significant difference.

Was there unfulfilled demand among irrational females for a conspiracy... (read more)

One thing I'm sure it's not is the example in #6 - that men are more into self-improvement. My wife is involved in the female self-improvement community, and there are endless workshops, conferences, books, etc.

What is different is the approach - it's on things like becoming more integrated, more aware of cultural prejudice, more aware of the impressions you give other people, more in touch with your body, avoiding repression. Not how to apply Bayesian reasoning to ordinary life. (I know that's a caricature of LW, but you get the idea). Women, on avera... (read more)

The gender ratio at physical meetups, while still unbalanced, seems noticeably better than the visible gender ratio among active commenters on the Internet.

That part is perfectly predictable. Men are less deterred than women by lack of face-to-face contact in relationships. Film at eleven.

5Nanani15y
I may just be a strange person, but lack of face-to-face contact is a Feature for me, not a Bug. Simply put, I find it easier to connect to a person's ideas and judge them on merit when my senses aren't throwing in distracting tidbits that often make me want to run away (such as "he is looking at me funny" or "he smells bad"). This is also the reason I prefer text to video blogs, emails to in-person meetings, and so on.
3mattnewport15y
Those examples suggest you're hanging out with entirely the wrong sort of people.
7Nanani15y
No no, I very explicitly do not hang out with the sort of people that elicit that sort of response. The problem is that the person eliciting it might not in fact be deserving of it, but I can't get far enough to find out.
0PhilGoetz15y
There were, I think, 7 OB men and no OB women at my OB meetup. (There were several women, but they had nothing to do with OB.)

I don't really know what the reason for the gender imbalance is, though I suspect reasons 4 to 8 all play a part, but I think it's highly likely that if you could find explanations for the gender imbalance in undergraduates studying math, physics and computer science, among sci-fi fans, programmers and libertarians and within the classic works of philosophy then you'd have sufficient explanation.

The fact that this question has been debated in all those areas for many years and we don't have very good answers suggests that it is not easy to answer. I think ... (read more)

You're brave to even touch this topic.

The obvious evolutionary psychology hypothesis behind the imbalanced gender ratio in the iconoclastic community - the atheist/libertarian/technophile cluster - is the idea that males are inherently more attracted to gambles that seem high-risk and high-reward; they are more driven to try out strange ideas that come with big promises, because the genetic payoff for an unusually successful male has a much higher upper bound than the genetic payoff for an unusually successful female.

I don't know if that's a factor, bu... (read more)

3Eliezer Yudkowsky15y
You're right, this is puzzling. Has it only been true for an evolutionarily short amount of time? Is seeking members of the opposite sex who share these kinds of interests a recent cultural invention? Is the claimed preference not as strong in reality as people think?

I would say both.

For much of our evolutionary history the idea of a consumption partner rather than a production partner would have been an unaffordable luxury. Desirable properties in a mate were primarily those that would support survival and reproduction.

I think the claimed preference is also weaker in reality than people think. This is a common theme in the seduction community. What people are actually attracted to is not necessarily what they say they are looking for - sexual attraction is not based on a conscious rational weighting of positive and negative attributes.

0taryneast13y
Really interesting link, and I'd never thought of it that way before either. Also - extra points for being the very first comment referencing the seduction community that actually provided a net plus to the conversation (regardless of gender) ;)
0MBlume15y
Thank you for the link -- I'd never seen that distinction made before and I suspect now I'm going to see it everywhere.
3Jack15y
Yeah, I think its only been a win for a short time. Maybe, for most of history hierarchy within the genders has mattered a lot more for sexual selection. And your position in the hierarchy is usually determined by your success at gender specific roles. So if you want to be the the alpha male you need to be really good at hunting and so the best hunters win. Similarly, power in the female hierarchy was dictated by things like child rearing and social knowledge. So those that focused on that won.
1MBlume15y
It seems like that would simply set up a sort of elastic restoring force opposed to whatever force is causing (for example) men to like video games, and you wind up at a sort of equilibrium. Presumably that's what we see around us.
-1SforSingularity14y
Why? I think that perhaps your reasoning is that you date someone based upon whether they have the same interests as you. But I suspect that this may be false - i.e. we confabulate shared interests as an explanation, where the real explanation is status or looks.

This of course just pushes the problem back a step, but isn't the breakdown in Myers-Briggs between Thinking and Feeling types something like 60:40 for men and 30:70 for women? Mightn't this have something to do with it?

Computer scientists are very highly represented here; a show of hands on IRC found more than half had some CS background. This site is particularly appealing to the CS mindset, so that's not so surprising, but it means that Less Wrong inherits the same massive gender imbalance that computer science has. Of course, this only pushes the question one step away, to the reasons why CS has a gender imbalance; but that's a question that's already been studied, with many hypotheses put forth.

i am a female and have been following OB and LW for about 3 months in googlereader, really liking it, although i never comment (how can i improve on elezier's genius?). may not matter, but i do work in IT. at any rate, i was compelled to register with this site since i wonder if the LW "group" doesn't contain more females than we think. Is the definition of a "member" anyone who consistently reads this blog, regardless of registration status, or anyone who has taken the extra 15 seconds to register, or those registered members who end u... (read more)

I've noticed strong female representation (where I least expected to find it) in The Skeptic Zone,an Australian skeptics group. The feeling I get of that community (even just as a podcast lurker) is that it's much more lighthearted than LW/OB. Whether that makes any difference to sex ratios, I don't know.

For most of the time I've listened to the podcast, there's been regular strong contributions from females. My gut feel would have been that having good female role models would encourage more female participation, however I just did a quick eyeballing of the Skeptic Zone's FaceBook fans and it looks typically about 5:1 biased to males.

3[anonymous]15y
Which came first? The chicks or the egg?
0MBlume15y
nice
1MBlume15y
Skepchick is also notable, I think.
1taryneast13y
Hmm... sometimes a "girls division" of a group can be a net plus. "linuxchix", "geekgirls" and "devchix" are also strong, enjoyable groups that tend to hang off the larger mainly-male groups, to the benefit of all. Of course, the opposite is also sometimes true... but would it hurt to start up, say, "LW girls" or similar? Anecdote: the annual linux.conf.au that was held in Sydney a couple of years back had the highest female:male ratio of any linuxconf.au so far (1 in 10), and that was due to the extremely strong support from the local linuxchix chapter. It took a local hero (Pia Waugh) and friends to make it happen... but it did help

This touches on something that I've been thinking about, but am not sure how to put into words. My wife is the most rational woman that I know, and its one of the things that I love about her. She's been reading Overcoming Bias, but I've never been completely sure if its due to the material, or because she's a fan of Eliezer. Its probably a combination of the two. In either case, she's shown no interest in this particular group, and I'm not sure why.

I also have a friend who is the smartest person and the best thinker that I've ever met. He's a practicing r... (read more)

Did you know that according to the last survey females (sex at birth) on LessWrong have a higher IQ with p=0.058?

Irresponsible speculation alert: people join LW because they dig the ideas and/or because they dig the community. The ideas are more enticing for people with higher IQ, the community is more enticing for.. guys. Thus, at equal levels of IQ more women will be filtered out because they feel (on average) less comfortable with the community.

Like I said, I don't assign the above explanation an overwhelming epistemic status, but I do think that the IQ... (read more)

1Vaniver9y
Eh, check the SAT scores. The difference is 0.14 points favoring men on the 1600, 29 points (about a tenth of a standard deviation) favoring women on the 2400, and .44 points (about a tenth of a standard deviation) favoring women on the ACT, and 3.7 points (about a quarter of a standard deviation) on the IQ self-report. I wouldn't trust the IQ numbers enough to rest an argument on that point. I would also state the argument a bit differently--it's not that at equal levels of IQ more women are filtered out than men (in which case the IQ distributions of the two would only be different matching any underlying population differences), but that the IQ filtering effect is stronger for women than men (or, stated symmetrically, less women are filtered out at higher IQs than lower IQs).

The obvious evolutionary psychology hypothesis behind the imbalanced gender ratio in the iconoclastic community is the idea that males are inherently more attracted to gambles that seem high-risk and high-reward; they are more driven to try out strange ideas that come with big promises, because the genetic payoff for an unusually successful male has a much higher upper bound than the genetic payoff for an unusually successful female. ... a difference as basic as "more male teenagers have a high cognitive temperature" could prove very hard to ad

... (read more)

Exposing yourself to karma judgements is similar to asking someone out on a date, or otherwise risking rejection. Men have to do this all the time; I think a typical man has to approach or flirt with over 100 women just to get 1 date. Women don't have to do it, and so don't get used to doing it.

5steven046115y
Karma-based explanations don't explain why we saw the same gender imbalance on OB.
3hhadzimu15y
Exposing yourself to any judgments, period, is risky. The OB crowd is perhaps the best-commenting community I've come across: they read previous comments and engage the arguments made there. How many other bloggers are like Robin Hanson and consistently read and reply to comments? Anyway, as a result, any comment is bound to be read and often responded to by others. There may not have been a point value attached, but judgments were made.

I was introduced to LW with a link and an endorsement that probably appeals more to the little boy in me than the little girl in others: "it's like martial arts for your mind."

Any thoughts on a 5 second sales pitch for women?

Perhaps women are less underrepresented on Less Wrong than may at first appear, and men are more likely to comment for some reason. Or perhaps women are less likely to choose visibly feminine usernames.

I think it's pretty frequent on the Internet, especially among those that have been around since the early-to-mid 1990s, to assume that everybody online is male by default until proven otherwise.

Being one of those, my first guess would be that the LW audience is > 90% male, but I'd love to see the results of an anonymous survey...

3taryneast13y
Less wrong survey is here and the results are:
2MichaelGR13y
Thank you. Goes to show that nothing goes away on the internet; I posted the parent comment 2 years ago.
2taryneast13y
Yeah - I saw it was an old comment... older than the survey. I figured I'd post it - in case anybody who came by later wanted to know.
2MichaelGR13y
Very cool of you. At least one person noticed!
0Grognor12y
(Just pointing out to any futureland readers that the situation has changed slightly.)
0JGWeissman15y
http://xkcd.com/322/
0Document7y
Did you mean to post that somewhere else?
-3taryneast13y
:) ah xkcd... much, much love!

higher cognitive temperature

That sounds like something very similar to what the Big Five model of personality calls Openness (see here).

(By the same logic that if we wanted more Japanese rationalists we might encourage potential writers who happened to be Japanese.)

BTW, in the 2012 survey there were zero Japanese, one Chinese, and two Koreans. Any idea what's going on?

Hypothesis 8, male variance in IQ, is irrelevant to the extent that this site is about rationality, not IQ. Whatever IQ tests measure, it is neither instrumental nor epistemic rationality. See What Intelligence Tests Miss: The Psychology of Rational Thought by Keith E. Stanovich for extensive discussion of this point. Even if there is male-female variance in IQ, that does not imply a male-female variance in rationality.

6Articulator9y
Pretty sure that the average IQ on LessWrong is above the mean, though. Therefore, a group with higher variance is more likely to have member in LessWrong. The causality of that statement is atrocious, but I think the overall picture should still come through.
[-][anonymous]9y00

Did you guys know that according to the last survey females (sex at birth) on LessWrong have a higher IQ?

[This comment is no longer endorsed by its author]Reply

Does anyone have any unbiased statistics on gender in workforce, career choice, education, and any other relevant statistics?

0taryneast9y
curious: how would this help?

On this hypothesis, male writers end up with mostly male readers for much the same reason that Japanese writers end up with mostly Japanese readers.

Because they can speak japanese?

4Swimmer963 (Miranda Dixon-Luinenburg) 11y
Or because they share basic cultural assumptions: how parents ought to treat their children, how children out to treat their parents, how teenagers ought to decide what to do as adults, how strangers ought to behave towards other strangers, etc. Japan and the USA have definite cultural differences, the most basic of which, massively simplified, is individualism versus collectivism as a society. This makes fiction written by Japanese authors and set in Japan seem alien to readers from the USA; in an appealing, artsy, interesting way maybe, but still alien. I can easily see such a mechanism operating with the cultural differences between males and females who both live the USA (or in Japan) could have a similar effect, making male-oriented fiction feel a bit odd and alien to girls who read a lot of chick lit because it's what their mothers and sisters and friends recommend all the time. I can see this transferring over, more subtly, to differences in styles of non-fiction writing and blogging. I don't know if it actually does, but it might.
0taryneast9y
It does (#justonedatapoint)

it looks like an easy way to get some karma would be to create a female username and post in this thread. If we say that women's comments in a thread about how to attract more women are more valuable we're making some unstated assumptions.

5AnnaSalamon15y
The profusion of upvotes did make me feel a bit odd, as though I'd wandered into a discussion with a tone of politics and random applause instead of collaborative thinking as I'm used to. But discussion here has nevertheless been a lot better than discussion on such mind-killing topics usually is (a testament both to LW and to EY's careful post). And I agree with mattnewport's point about erring on the side of being welcoming, especially for those who said they are nervous and new.
4pjeby15y
What makes you say that? At the present moment, only one female comment in this thread is rated higher (20) than my highest-rated comment in this thread (17). My second-highest rated comment (11) is higher than most other female comments in the thread. There are plenty of other male comments with comparably high scores, in the 9-12 range. It appears that people are simply voting up more in this thread in general, not merely to persons with female handles. My personal guess would be that this is because the air is getting cleared on some topics of general interest that nobody has felt comfortable expressing prior to this point, and the refreshment of hearing those things (and a few good proposed solutions) is encouraging upvotes. (Of course, there could also be a priming effect of focusing on stereotypes around social support and being a welcoming community in general, which would also skew things more towards upvotes and away from downvotes...)
0mattnewport15y
You may have a valid point but since a common theme seems to be that newer posters do not feel welcomed or that their input is appreciated I believe people are making an effort to be more generous with up-votes in this thread. I've generally been quite parsimonious with up-votes and comments here and in the welcome thread have made me feel that I should bias myself more in favour of up-votes over down-votes. Since this is a thread about how and why females are underrepresented on LW their input is particularly relevant and so higher karma scores do not necessarily imply any further bias. In conjunction with a hypothesized general realization that more up-voting, particularly of new commenters, would be of benefit to the community I think it's possible to explain the high comment karma without postulating further bias. I'd be interested to know what you think the unstated assumptions are though.
0steven046115y
Can you name an example of a wrong assumption that we might be making if we're saying this?

Could some of this be resolved through technology?

Imagine a voting system which takes into account the gender of the person voting, as well as the gender of the person viewing the page. A woman reader's view might place higher value on women's votes, relative to men's, such that maybe a single downvote from another woman will count much farther towards making a comment invisible than several upvotes from a men.

(with maybe a twiddle somewhere that says something like "show me the men's view" "show me the women's view" "show me both views, highlighting differences" "show me both views, ignoring differences")

3aausch13y
I'm a bit confused by the downvotes. Did I miss something? I figured that my suggestion, or some approximation in the same solution space, would both provide useful information about the cause of the gender imbalance, and tools to try and address it.
1AdeleneDawner13y
Collecting information on the voting patterns of different categories of people might be useful. Having different things shown to different people based on what category they're in, though? Ew, no.
0aausch13y
Are you opposed to it because it's divided along gender lines? Would you be more receptive to it if it was divided along, say, age lines, or proficiency in rationality lines?
3AdeleneDawner13y
If proficiency at rationality could be shown to be a single skill or a set of skills that are consistently improved on in an even way (so that there aren't people who are very good at one kind of rationality and very bad at another), and if we had a reliable way of measuring that trait, that might be usefully used to weight votes, though it wouldn't make sense for low-rationality people to see scores based on the votes of other low-rationality people rather than scores based on the votes of high-rationality people. I'm not confident of either of the premises, though. In the other cases, no, it's still a bad idea.
0aausch13y
I'm trying to understand where the bad is in this idea. Are you maybe opposed to details of the implementation? Would you think the idea is bad if the option to filter out results is opt-in and explicitly stated? For example, offer users a "only use votes from teenagers when displaying data on the site" option, which they can enable or disable at will.
0AdeleneDawner13y
If it's opt-in, explicitly stated, and not limited to groups that the user has declared themselves to be a member of, there's probably no harm in it - it'd just be another kind of information. Your original suggestion was missing some of those features, most notably the opt-in option.
1wedrifid13y
I upvoted for practical thinking. Some of the complaints made about this kind of topic would, in fact, be resolved by the solution you propose. That said the overall effect of implementing the change would be detrimental.
0thoughtengineer12y
Although I would be in favor of increased statistics on comments / votes in order to allow the community as a whole to analyze what writing styles, etc. favor different subgroups, I think having a voting system that prioritizes different voters simply due to characteristics they display isn't a healthy way to handle this issue, as IMHO it would lead to more "us vs. them" dichotomous thinking instead of viewing us as a group of individuals focused on understanding and developing rationality in our lives and others. After all, the question is not, at least in my mind, how can we get more people onto LessWrong and not offend them when they get here, but rather to culture a intellectual pocket that fosters open discussion and self-improvement. If some people are offended when their ideas are not accepted (as long as it isn't rudely accomplished) I don't think that's something that we should address through the voting system, as rationality is at least partially dependent on not getting besotted with a single idea or concept, regardless of its value and accuracy.

I'll expand on the controversial stance as to why this is. It's obviously extremely complicated and I can't really do it justice in this specific comment. However I will try to give a cursory explanation. I personally tend to think that rational thinking may in part be genetically encoded in the brain and is a trait that may be somewhat distinct from aspects of general intelligence. I think its very likely that their has been differential selection pressures on male and female brain's over the course of evolution. There has been a lot of evolutionary ... (read more)

4Jack15y
There is no way. I mean no way, that there was any significant selection pressure in favor of reading more and a reading more salient works. For the vast majority of human history there were no written works to seek out. Even once writing was developed it was used almost exclusively for book keeping. Then people wrote down myths. I like the legend of Gilgamesh as much as the next guy but reading it never conferred an evolutionary advantage on anyone. Were our ancestors seeking ancient hunting manuals?
2Mike1239015y
Well here's a paper about the changes in surnames in Britain between 1600 to 1851. This is recent evolutionary selection. http://www.econ.ucdavis.edu/faculty/gclark/Farewell%20to%20Alms/Clark%20-Surnames.pdf Wealthier people quickly out bred poorer people. I think its highly probable that genes were selected for to be able to better seek out and absorb written material. Being able to learn information has a huge fitness benefit to propagating your genes. There was plenty of scientific reading material available at this time (1600-1851) that could have conferred a fitness benefit. Any factual information you learn about the world (through reading) can improve your odds of avoiding death. Death was very common in the recent past. http://www.udel.edu/educ/gottfredson/reprints/2007evolutionofintelligence.pdf
5Jack15y
Its an interesting study but I don't think it proves your point. Wealthy people out bred poor people in a span of 10 generations in England. But I have no particular reason to think that the reason the rich outlived the poor was their ability to read. Access to better food, more sanitary living, less stress all indicate higher survival rates. Its not surprising the the children of the wealthy were more likely to survive into adulthood. Now maybe we think the rich were better readers to begin with and the gene spread that way. But we have no particular reason to believe this either- or rather we might have reason to believe the rich were better readers but we have no reason to believe this advantage was genetic.
2Mike1239015y
Being able to learn and absorb written material allows a person to create more wealth. This is presumably why people go to school at all. Being able to accumulate more wealth as a result of learning allows a person to have increased reproductive fitness. So when you say "Access to better food, more sanitary living, less stress all indicate higher survival rates", these things could be the result of learning information. Like if you learned how to build something from a book that could improve your surrounding living conditions. Of course this could be highly variable depending on an individual person's evolutionary past. Genes have been correlated with reading ability. http://www.physorg.com/news142091390.html I would imagine their could have been selection pressure on these genes in the recent past. See here for more info. http://jmg.bmj.com/cgi/content/full/44/5/289
2taryneast13y
I think you may have your cause and effect backward here. in the 1600's wealth is the cause of people going to school - not the other way around. The vast majority of people had no access to schooling at all (unless they joined the clergy and thereby their line ended). Accessible schooling is a very modern phenomenon. You may thereby be confusing correlation with causation. Reading and wealth are correlated because the latter causes the former, rather than vice versa. Genes are correlated with reading simply because wealth is correlated with the ability to multiply and support many descendants. Therefore we should not be surprised that reading correlates with genes, any more than that there is likely a correlation between genes and wearing expensive, fashionable clothing.
2Jack15y
Well the second article claims that ev psych cannot explain reading ability and that this is-- wait for it!-- evidence of intelligent design. Pretty sure thats neither of our advocacies. Being able to learn and absorb written material allows a person in our economy to create more wealth. Its not clear to me at all that that has always been the case- at least in degrees high enough to exert enough selection pressure over only a handful of generations to account for a reading ability gender gap. Your average man in 17th century England doesn't learn to read because his best opportunity to increase his earning is to put in an extra hour on the farm. Or apprentice as a black smith. Or become a sailor. Those with the means might go to school and become lawyers or doctors- but they were already rich. Its not like they had academic scholarship or pell grants. Its also recalling that in this time period the literacy rate was considerably lower than it is today. And thats not because vast majorities in Europe didn't have the genes to read during the 18th-- its because they weren't taught to read. That means any selection that was happening was only happening within the small subset of the population that was given the opportunity to learn.
0Mike1239015y
http://books.google.com/books?id=DyMjW21HwHwC&pg=PA89&lpg=PA89&dq=reading+literacy+england+1600&source=bl&ots=SL1ct7yRfW&sig=0Hz6txLaE3_51PwzxFQTxSdEka4&hl=en&ei=4NHmSdjPF8LgtgeVvvCVBg&sa=X&oi=book_result&ct=result&resnum=1 30 percent of english men could read in 1600. I would say that is a considerable amount. Only 10 percent of women could read at that time. By 1700 50% of men could read. So I would bet that the 30 percent that could read in 1600 out bred those who couldn't. It's possible that the increase in reading literacy was partly genetic in origin. Rational/logical/analytical abilities could help with learning. There are obviously a lot of complex interactions at work in our evolutionary past that I think we are just begining to understand.
3Jack15y
All those older literacy statistics, as it says in the very next sentence in your source, are based on the ability to sign one's name. Thats not exactly evidence of superior analytic reading and writing skills. There are perfectly ordinary, non-genetic reasons for the increase in literacy. For one, doesn't it seem strange to you that literacy rates increased all across Europe at roughly the same time? Shouldn't some nations have remained illiterate until they interbred with the literate ones? Instead, literacy correlates perfectly with widespread economic and cultural changes. How exactly did people start reading in Europe at all? For a long time it was only the monks who could manage it and they weren't passing their genes on. Also, from a quick google it looks like the dyslexia gene set, which is the one you previously identified as evidence for the genetic basis of reading, is autosomal, which means its spread should be equal among males and females.
0Mike1239015y
About the literary statistics, it specifically says "These figures may be pessimistic because reading was taught before writing at school". So the figures may actually underestimate the literacy rate. I don't have the time to found other sources. I'm not saying the increase in literacy rates was all genetic in origin. The fact that literacy increased on a societal scale probably had to do with new inventions that made it easier to publish and distribute written material (I'm not sure of the whole history to be honest so that is somewhat of a guess). However these societal changes may have also led to genetic changes as well. Read the "10,000 year explosion" for more examples of how societal changes lead to gene frequency changes that led to more societal changes. Some animals have rudimentary reading capabilities. Being able to read may just be an offshoot of some other brain process (reading facial expression for example). So evolution is often good at co-opting brain processes for alternative uses. With a wider distribution of reading material it is possible that evolution selected gene frequencies that altered reading ability (speed of reading, logical analysis, etc.) I don't think the it matters if the dyslexia gene is the same in females versus males. The phenotypical effect of the gene on the host (male vs. female) would likely be different. The fact that men and women have different hormones could alter the expression of any single gene (for example). The phenotypical effect of any gene in an organism is going to depend on a multitude of factors (like environment, other genes). That's all I can say for now.
0thoughtengineer12y
Agreed. Furthermore, I think that we should consider the fact that even if there was significant selection pressure for the ability to be literate (I would argue that this is most likely not the case) that there is insufficient evidence that this pressure could result in a significant social difference so quickly. Furthermore, the converse appears to be currently true: the more educated a women is (I would assume this relates to reading comprehension, although not necessarily so) the expected number of children she will have generally decreases (http://economix.blogs.nytimes.com/2010/06/25/educated-women-opting-for-motherhood/).